Kaplan Cumulative: Psych

Lakukan tugas rumah & ujian kamu dengan baik sekarang menggunakan Quizwiz!

A 26-year-old man is brought to the hospital by his family after sitting in his room with the lights out and the door closed for 2 days. He has not eaten over this time. About a week ago, the family noticed the patient becoming increasingly agitated and paranoid about cars driving by on the street in front of their house. He covered the windows of his bedroom with newspaper and unplugged his radio and television. At night, he was heard pacing in his room and talking to himself. Although he had taken olanzapine (Zyprexa) after a psychiatric hospitalization about 6 months previously, the family reported that he threw away the medication about a month ago. On admission, the patient is sitting in a chair with his head hung low. He is disheveled and malodorous, after having urinated on himself several times over the past 2 days. During physical examination, the patient appears to be awake, but firmly resists any attempts to be moved. He does not follow instructions, and the nurse was unable to move his arm to obtain a blood pressure measurement without assistance. Which term would best describe this patient's resistance to being moved? A. Akathisia B. Cataplexy C. Echopraxia D. Negativism E. Stereotypy

A 26-year-old man is brought to the hospital by his family after sitting in his room with the lights out and the door closed for 2 days. He has not eaten over this time. About a week ago, the family noticed the patient becoming increasingly agitated and paranoid about cars driving by on the street in front of their house. He covered the windows of his bedroom with newspaper and unplugged his radio and television. At night, he was heard pacing in his room and talking to himself. Although he had taken olanzapine (Zyprexa) after a psychiatric hospitalization about 6 months previously, the family reported that he threw away the medication about a month ago. On admission, the patient is sitting in a chair with his head hung low. He is disheveled and malodorous, after having urinated on himself several times over the past 2 days. During physical examination, the patient appears to be awake, but firmly resists any attempts to be moved. He does not follow instructions, and the nurse was unable to move his arm to obtain a blood pressure measurement without assistance. Which term would best describe this patient's resistance to being moved? A. Akathisia B. Cataplexy C. Echopraxia D. Negativism E. Stereotypy Show/hide explanation The correct answer is D. This patient is exhibiting the catatonic symptom known as negativism, which is a motiveless resistance to all attempts to be moved or to all instructions. Signs of catatonia include stupor, negativism, rigidity, posturing, mutism, stereotypies, mannerisms, waxy flexibility, and catatonic excitement. Catatonia may be associated with schizophrenia (catatonic type), mood disorders (with catatonic features), or general medical conditions. Akathisia (choice A) is usually classified as an extrapyramidal side effect of antipsychotic medications. It is characterized by a subjective feeling of muscular tension (an inner sense of restlessness) that can cause distressing restlessness, pacing, or repeated movements. It may be mistaken for psychotic agitation, and thus inappropriately treated. Cataplexy (choice B) is a temporary loss of muscle tone and weakness precipitated by a variety of emotional states. It is most characteristically associated with narcolepsy. Echopraxia (choice C) is a pathologic imitation of the movements of one person by another. It can be seen in catatonia, delirium, dementia, and other disorders. Stereotypy (choice E) is a repetitive fixed pattern of physical action, movement, or speech. It may be seen in catatonia. This patient does not currently exhibit stereotypies. Signs of catatonia include stupor, negativism, rigidity, posturing, mutism, stereotypies, mannerisms, waxy flexibility, and catatonic excitement. Catatonic negativism occurs when patients refuse all instructions. Catatonic rigidity refers to immobility and resistance to all movement. Catatonia may be associated with schizophrenia (catatonic type), mood disorders (with catatonic features), or general medical conditions. Catatonia is the "umbrella" term for akathisia, cataplexy, echopraxia, negativism, and stereotypy.

A mother brings her 8-year-old son to a mental health provider for new-onset enuresis. A prior workup to determine a medical cause was negative. In conversation, it seems that the enuresis started following parental arguments and separation. The boy wets himself at least twice a week and feels upset about it, refusing to go for a sleepover at his friend's house. Which of the following is the most commonly used treatment for this condition? A. Behavioral therapy B. Interpersonal therapy C. Pharmacotherapy D. Psychodrama E. Psychotherapy

The correct answer is A. Behavioral therapy is the most frequently used treatment in children who have enuresis. Dry nights are recorded on a calendar and rewarded with a star as a gift. The buzzer and pad apparatus is used whenever other therapies fail. Enuresis is defined as micturition that is normal but occurs at an inappropriate time or socially unacceptable time or place, typically at night. This enuresis occurs at a time after bladder control should be established, which is typically after age 5. Nocturnal enuresis is also known as bedwetting. Primary enuresis means that the child never had a period of dryness longer than 6 months, and secondary enuresis occurs after a child had a non-bedwetting period of at least 6 months. Differential diagnosis includes diabetes, urinary tract infection, or emotional problems. Treatment is based on the subtype and involves reassurance, management of contributing medical conditions, education, and behavioral measures. Alarm therapy can be used if the abovementioned treatment strategies do not work.

A 52-year-old woman is attending her 18-year-old son's funeral, when she cries out, "Damn you, Lord, for stealing my baby from me!" During the next several days, she has several outbursts of crying spells and extreme anger at God, and questions her religious faith. After a few days, the crying spells cease. She has never been diagnosed with a psychiatric disorder in the past. She is well supported by friends and family, and she denies any suicidal ideation. She took 2 days off from work following her son's death, but even though she thinks of him often and sometimes has difficulty concentrating, she has returned to work. Which of the following is the most likely diagnosis? A. Bereavement B. Brief psychotic disorder C. Major depressive disorder with psychotic features D. Panic disorder E. Paranoid schizophrenia

The correct answer is A. Bereavement, or normal grief, is often characterized by many of the same characteristics of depression, including sadness, tearfulness, loss of appetite, poor sleep, and diminished interest in activities that used to bring pleasure. Of importance in differentiating grief from major depressive disorder (choice C) is that people who have grief have symptoms that are time-limited, whereas people who have major depression cannot ever imagine feeling better. People who have bereavement or grief reaction have an identified stressor that precedes the depression, whereas the majority of patients who have major depression have endogenous depression without any outside identified stressor that leads to the change in mental status. Bereavement or normal grief reaction patients have symptoms consistent with major depression, but these mental status changes are secondary to a known event. Normal grief reaction patients may hear the voice or see the loved one who has died and this is considered to be within normal limits as part of this clinical syndrome. Grief reaction patients should have resolution of symptoms within 4-6 months of the acute stressor, and if symptoms continue longer than this time period, the patient should be evaluated for major depression.

A 29-year-old woman is brought to the hospital by her husband. She has not slept in several days and cleans the house, drinks wine, and listens to loud music in the middle of the night. She spent $2,000 on a shopping spree over 4 days and decided to change her career and start a private business. In the interview room, she talks incessantly, giggles with the nurse, and unbuttons her blouse to show her newly bought underwear. She has always been cheerful, and has had short periods of time when she was more energetic, but never like this. She denies the use of street drugs, and her urine drug screen is negative. Which of the following is the most likely diagnosis? A. Bipolar disorder type I B. Bipolar disorder type II C. Borderline personality D. Cyclothymia E. Schizophrenia paranoid type

The correct answer is A. Bipolar disorder type I includes full-blown episodes of mania with erratic and disinhibited behavior, grandiosity, overextended activities, poor frustration tolerance, and vegetative signs, such as increased libido, excessive energy, decreased sleep, and weight loss. It also includes episodes of mixed mood and major depression. Bipolar I disorder is a chronic and recurring mental illness that is marked by alternating periods of abnormal mood elevation and depression that are a marked change from the person's baseline status of functioning. Depression symptoms predominate with at least one episode of mania that adversely affects functioning.

A 58-year-old woman arrives at her health care provider's office complaining of moderate anxiety. Which of the following drugs will help relieve her anxiety, with a minimum of unwanted sedative side effects? A. Buspirone (Buspar) B. Chlordiazepoxide (Librium) C. Lorazepam (Ativan) D. Trazodone (Oleptro) E. Zolpidem (Ambien)

The correct answer is A. Buspirone is a nonbenzodiazepine anxiolytic that is devoid of the sedative (or anticonvulsive and muscle relaxant) properties typically associated with the benzodiazepines. It is a partial agonist at 5-HT1A receptors, and it binds to both serotonin and dopamine D2 receptors. Because this is not a benzodiazepine, there is less chance of severe drug interactions if the patient drinks alcohol along with taking this medication. Buspirone is indicated for treating anxiety and functions in a similar role as benzodiazepines. It affects the GABA receptor. It is not likely to cause sedation or rebound phenomena when it is

A 27-year-old successful businesswoman has developed a fear of flying after an extremely rough landing. She is paralyzed with fear and unable to travel for business. Her health care provider tried giving her lorazepam (Ativan) to take during the flight, but it didn't help. She returns to the health care provider and asks if there is anything else that she can do to reduce her fear because she is not getting a promotion at work because she cannot travel. Which of the following is the most commonly used non-drug intervention for this disorder? A. Exposure therapy B. Hypnosis C. Insight-oriented psychotherapy D. Medication E. Supportive therapy

The correct answer is A. Exposure therapy, a type of cognitive behavior therapy, is the most commonly used treatment of specific phobia. The therapist usually desensitizes the patient by a gradual exposure to the phobic stimulus. Relaxation and breathing control are important parts of the treatment. Obsessive-compulsive disorder has its first-line therapy with cognitive therapy that involves exposure and response prevention. Patients are given a hierarchy of symptom triggers and each patient is then encouraged to expose him or herself to the trigger. The patient is then encouraged to refrain from engaging in compulsive rituals. As the patient improves, more intense triggers are targeted until the patient is able to face these triggers without having the compulsive behavior employed.

A 41-year-old man with a long history of schizophrenia, paranoid type, has been on the inpatient unit for almost 2 weeks. This is his third admission in the past 6 months, and each time he seems to be less responsive to treatment. In the past he has been on typical antipsychotics and then risperidone (Risperdal), with limited success. His family is supportive and makes sure he takes his medication. Given his most recent poor response, which of the following is the most appropriate next step in treatment? A. Clozapine (Clozaril) B. Droperidol (Inapsine) C. Electroconvulsive treatment D. Lamotrigine (Lamictal) E. Topiramate (Topamax)

The correct answer is A. Clozapine is an atypical agent that has been proven to be more effective than conventional antipsychotics in the treatment of patients who have resistant schizophrenia. It exerts an antagonistic effect on D1 and D4 receptors, as well as on the alpha-adrenergic, histaminergic, serotonergic, and cholinergic systems. It may be effective in the treatment of resistant schizophrenia in the first 6 weeks. It produces troublesome side effects, however. Its most serious side effect: agranulocytosis. Cardiomyopathy and myocarditis are also known side effects of this medication. Clozapine requires regular blood count monitoring because of this serious but rare side effect. Clozapine (Clozaril), an atypical antipsychotic agent, is useful as an antipsychotic for patients who are resistant to treatment of schizophrenia. Because of the rare but serious side effect of aplastic anemia and cardiomyopathy, this agent should be used only after other treatments have failed and not used as a first-line antipsychotic.

A 39-year-old businessman with no prior medical problems is rushed to the emergency department following the sudden onset of dizziness, shortness of breath, and palpitations. His blood pressure on admission is 190/110 mm Hg, pulse 124/min, and he is diaphoretic. His wife says that his behavior has changed over the past couple month since he became CEO of his company. He has become moody. At times he seems energetic, euphoric, or irritable; then he seems "to be down" for no reason. He just returned from one of many business meetings and again spent more money than ever before. The patient is smiling inappropriately and denies any alcohol or drug abuse. Which of the following will most likely be found on a urine drug screen? A. Cocaine B. Heroin C. Nicotine D. Organic inhalants E. Phencyclidine

The correct answer is A. Cocaine intoxication is characterized by sympathic stimulation, including tachycardia, hypertension, and sweating. The mood is elated and euphoric while intoxicated, and there is restlessness and pressured speech. Psychotic symptoms can occur with prolonged use. Cocaine intoxication leads to short-term states of autonomic arousal. Chronic use can harm the heart with scarring and myocardial hypertrophy (to overcome high blood pressure and systemic vascular resistance). Cocaine intoxication can lead to lethal arrhythmias and sudden death. Prolonged QT intervals can also occur as the normal heart repolarizing process is upset. Chest pain related to cocaine use is a common complaint seen in emergency medicine practices but rarely causes acute myocardial infarction unless there is pre-existing pathology. Mental status changes can include "excited delirium" with an agitated confusional state that can be associated with lethal hyperthermia.

A 49-year-old man comes to a mental health care provider for depressive symptoms. He talks about his past psychiatric problems. He also mentions he is an alcoholic and has been taking disulfiram (Antabuse) for some time to keep sober. He once tried to drink after taking the drug and ended up being terribly sick. Which of the following principles best describes the particular treatment for alcoholism? A. Conditioned avoidance B. Extinction C. Flooding D. Positive reinforcement E. Reciprocal inhibition

The correct answer is A. Conditioned avoidance is a term that describes the pairing of an unpleasant stimulus with the stimulus that causes maladaptive behavior. This aversive conditioning technique makes it uncomfortable or painful, in order to have the patient avoid the behavior that is trying to be eliminated. Extinction (choice B) requires the removal of the reward for inappropriate behavior so that maladaptive behavior decreases. It is often used in child psychiatry with patients who have behavioral problems. Flooding (choice C) is a therapeutic technique in which a patient is exposed to the feared situation without the possibility to escape. This experience is stressful and it must be done in a supervised and controlled manner. Positive reinforcement (choice D) happens when a subject is rewarded for manifesting desired behavior. Reciprocal inhibition (choice E) happens when a response that is antagonistic to the undesired behavior is paired with the behavioral response (e.g., relaxing along with anxiety provoking stimuli). FDA-approved medications for the treatment of alcohol dependence act on the neurobiological systems that are believed to be responsible for alcohol craving and reward centers. Naltrexone (Revia ) is an opioid antagonist effective in patients who have a family history of alcohol dependence and for those who have significant alcohol craving. Acamprosate (Campral) appears to stabilize glutamate and GABA neurotransmitters and is approved for maintenance of abstinence of alcohol. Disulfiram (Antabuse) blocks the metabolism of alcohol, which results in the buildup of toxic acetaldehyde. This lack of alcohol metabolism leads to flushing, nausea, diarrhea, tachycardia, and hypotension. This medication is taken on a daily basis and the threat of significant illness is one form of aversive conditioning to have a patient stop drinking alcohol. Intake of metronidazole (Flagyl) can also have this disulfiram-like effect.

A 6-year-old boy with mental retardation has recently been diagnosed with fragile X syndrome. His 9-year-old sister appears to be of normal intelligence but has symptoms of attention deficit hyperactivity disorder (ADHD). She has no significant medical conditions. What is the first test that is indicated in her workup for ADHD? A. Cytogenetic testing B. EEG C. Intelligence quotient (IQ) test D. MRI E. Urine for metabolic screen

The correct answer is A. Cytogenetic testing should be performed on sisters of males who are diagnosed with fragile X. Heterozygous females frequently have developmental and behavioral problems such as ADHD. They may also have borderline or mild mental retardation. The sisters of male children who have fragile X syndrome should receive cytogenetic testing for ADHD because heterozygous females may have similar behavioral and developmental problems as those who have ADHD. Female patients who have fragile X may exhibit learning disabilities and mental retardation.

A 25-year-old actress tends to behave seductively and with exaggerated charm to obtain continuous attention. She often puts on an overdramatized display of emotion but lacks genuine deep feelings. She craves control in her interpersonal relationships, often causing them to disintegrate. This patient most likely has which of the following personality disorders? A. Histrionic B. Narcissistic C. Paranoid D. Passive-aggressive E. Schizotypal

The correct answer is A. This is a typical description of histrionic personality disorder. Clues are seductive behavior, exaggerated charm, need for attention, overdramatized emotions, shallowness of feelings, and craving control. Patients who have histrionic personality disorder are attention-seeking and put themselves into position to become the center of attention. They may behave seductively and overdramatically. They also want to be in control of situations. Because this is a personality disorder, treatment consists of patient communication and relationship management along with psychotherapy for the patient to gain insight into his or her behavior.

An 8-year-old boy is brought to see a child and adolescent health care specialist because of his mother's complaint that he repeatedly states that he wants to be a girl. She also notes that he has always seemed to prefer girls' clothes and frequently gets upset in the mornings when his mother dresses him in typical boy clothes for school. His play activities are characterized by games more frequently enjoyed by girls, and he prefers to play with girls. When his mother reminds him that he is a boy and should act like a boy, he gets upset and sometimes has a temper tantrum. The boy has no significant past medical history, and physical examination is within normal limits. Into which of the following areas is this boy's difficulty best classified? A. Gender identity B. Intellectual development C. Sexual aversion D. Sexual identity E. Sexual orientation

The correct answer is A. Gender identity is a person's sense of maleness or femaleness. The formation of gender identity is based on many cultural influences, physical characteristics, and parental attitudes. The standard and healthy outcome in the development of gender identity is a relatively secure sense of identification with one's biological sex. Individuals may be diagnosed with gender identity disorder when the disturbance causes clinically significant distress or impairment in social, occupational, or other important areas of functioning. Patients suspected of having gender identity disorder should have subspecialty referral because of the complexities involved with its identification and treatment. Of the subtypes, transvestitism is most commonly encountered, whereas transsexualism is the least common. Disorders of gender identity occur when there is a dissonance between a person's biological sex and their sense of being male or female. Gender identity of childhood involves a persistent and intense distress about being assigned to a particular sex along with a desire to be of the opposite sex.

A 32-year-old man with a history of panic disorder who works as a salesman returns to his health care provider after several weeks of treatment with paroxetine (Paxil). Although he has noted significant improvement in his symptoms, he still notes residual anxiety when put into social situations in which he has to speak in public or become the center of attention. He has been taking the medication as prescribed and has not missed any doses. Given that he had some improvement on his current medication regimen, which of the following would be the most appropriate next step in management? A. Cognitive-behavior therapy B. Electroconvulsive therapy C. Medication change D. Psychoanalysis E. Seeing the patient more often for supportive psychotherapy

The correct answer is A. In many cases of panic disorder, effective treatment involves the use of cognitive behavior therapy, which incorporates exposing the patient to disturbing stimuli in an attempt to develop coping mechanisms in response to the stimuli. First-line therapy for social anxiety disorder is with cognitive behavioral therapy, which should be continued for at least 12 weeks. SSRIs are also approved as dual first-line therapy for this condition. SNRI agents are also indicated for treatment. Short-term benzodiazepine treatment can also be given until the SSRI or SNRI becomes fully functional.

A 40-year-old man is brought in for evaluation by the Coast Guard after the small plane he was piloting crashed into the ocean. The man's wife and 2 friends were also on the plane. The man has survived the crash with cuts and a broken arm, but he claims he has no memory of the crash or how he escaped the plane. He is also unable to explain how he got his life jacket on. His physical examination now is significant only for minor lacerations and a fractured right humerus, and he has no alteration in consciousness. A CT scan is normal. He is very upset that the fate of his wife is unknown, and he has nightmares for the next several nights while trying to sleep. Which of the following is the most likely diagnosis? A. Dissociative amnesia B. Dissociative fugue C. Dissociative identity disorder D. Factitious disorder E. Transient global amnesia

The correct answer is A. In this instance, an extremely stressful event has been followed by localized loss of memory or amnesia of circumstances surrounding the event, making the diagnosis of dissociative amnesia the most likely diagnosis. Dissociative amnesia is often accompanied by nightmares and anxiety concerning the event, both of which this patient also has. Dissociative disorder involves a period of detachment from self or surrounding experience as being "unreal" or "outside" of the person or self. This condition is associated with periods of time that the person cannot recall. Fugue states involve a person who leaves a geographic area and assumes another personality that is not consciously produced or occurs without knowledge of various personalities occurring within the person. Dissociative identity disorder is usually precipitated by a stressful situation and it is theorized that the alter identity is formed to escape from the stressful situation. Dissociative amnesia occurs when the affected person involuntarily represses the stressful memory but also has nightmares and anxiety over the stressful situation; this is not consciously produced.

A 49-year-old veteran comes to the health care provider to discuss smoking cessation. He was recently diagnosed with high blood pressure, hypercholesterolemia, and coronary artery disease, and he now realizes that it is finally time to quit. He has been a heavy smoker for years and finds it impossible to stop on his own, but his wife has heard about the use of bupropion (Zyban) for the treatment of nicotine dependence. After discussing the pros and cons of starting bupropion (Zyban), the patient agrees to give it a try. Which of the following instructions should be given to the patient about the use of this medication and its relationship to smoking cessation? A. He should decrease smoking after 5-7 days of treatment B. He should set a quit date after reaching the therapeutic dose of 450 mg C. He should stop smoking on day 3 after starting bupropion D. He should stop smoking right away E. His stopping date should be after 6 months of treatment with bupropion

The correct answer is A. Instructions to the patient regarding smoking cessation once medication treatment has started include gradual discontinuation of smoking and stopping smoking after 5-7 days of treatment. While the dose is titrated up after the first three days of therapy. Because bupropion (Zyban) is effective in cutting down cravings, the patient who is slowly titrating down smoking patterns will be more effective as the clinical effects of bupropion are seen.

A 33-year-old woman comes to the health care provider because of anxiety and irritability. She states that approximately 10 days before the onset of her menses, she goes from "feeling fine" to "feeling awful." She has frequent angry outbursts toward her husband and children, feels depressed and irritable, and cries easily. She also develops severe breast tenderness during this time. The symptoms resolve soon after the start of her menses and she feels fine until the symptoms start over again as the next menses approach. She has had this constellation of symptoms every cycle over the past year and feels that she can't go on like this anymore, as her job and family relations are suffering greatly. She has no other medical problems and takes no medications. Her physical examination is entirely normal. Which of the following is the most appropriate management? A. Fluoxetine (Prozac) B. Oophorectomy C. Primrose oil D. Progesterone E. Vitamin B6 (pyridoxine)

The correct answer is A. This patient has a presentation that is consistent with premenstrual dysphoric disorder (PMDD). This disorder is considered a form of premenstrual syndrome (PMS) with more severe emotional symptoms. Selective serotonin reuptake inhibitors (SSRIs) are the initial drugs of choice for PMDD or severe PMS. Fluoxetine (Prozac) and sertraline (Zoloft) are both approved for the management of this disorder. Patients who have severe PMS may be placed on a daily dose. Another approach that has been shown to be effective is intermittent therapy, in which the SSRI is administered only during the symptomatic phase. This method reduces the overall rate of side effects and is less expensive. Fluoxetine (Prozac) has the longest half-life of all the SSRIs, thus may be especially helpful in this disorder as missed doses are not as impactful. Premenstrual dysphoric disorder (PMDD) is associated with the cyclic occurrence of depression, anxiety, and emotional lability that occurs 1 week prior to menses and dissipates within 1 week after the completion of menses. These symptoms are most pronounced in the luteal phase of the menstrual cycle and may also include anger, irritability, and internal tension. Physical symptoms include breast tenderness, abdominal bloating, and headaches. To meet the DSM criteria, the patient must have ≥5 of the above signs or symptoms. Approximately 3-8% of menstruating women meet the criteria for this condition. SSRIs are the medication class of choice for PMDD. Alternative second-line options include calcium supplementation, pyridoxine, magnesium, and vitamin E.

A 49-year-old veteran comes to the health care provider to discuss smoking cessation. He was recently diagnosed with high blood pressure, hypercholesterolemia, and coronary artery disease, and he now realizes that it is finally time to quit. He has been a heavy smoker for years and finds it impossible to stop on his own, but his wife has heard about the use of bupropion (Zyban) for the treatment of nicotine dependence. After discussing the pros and cons of starting bupropion (Zyban), the patient agrees to give it a try. Which of the following instructions should be given to the patient about the use of this medication and its relationship to smoking cessation? A. He should decrease smoking after 5-7 days of treatment B. He should set a quit date after reaching the therapeutic dose of 450 mg C. He should stop smoking on day 3 after starting bupropion D. He should stop smoking right away E. His stopping date should be after 6 months of treatment with bupropion

The correct answer is A. Instructions to the patient regarding smoking cessation once medication treatment has started include gradual discontinuation of smoking and stopping smoking after 5-7 days of treatment. While the dose is titrated up after the first three days of therapy. Because bupropion (Zyban) is effective in cutting down cravings, the patient who is slowly titrating down smoking patterns will be more effective as the clinical effects of bupropion are seen. The recommended dosage of bupropion for smoking cessation is 300 mg/day given as 150 mg, 2x daily. Dosing should begin at 150 mg/day for the first 3 days and should be increased to 300 mg/day thereafter. Initiate treatment while the patient is still smoking and set a target date for smoking cessation after 5-7 days of treatment. Continue therapy for 7-12 weeks. If the patient has not reduced smoking by the 7th week, it is unlikely that he will quit during that attempt, and therapy should be discontinued.

A 27-year-old woman, gravida 2, para 2, comes to the health care provider to have her staples removed after an elective repeat Cesarean delivery. She also underwent a tubal ligation at the time of her Cesarean section. Her pregnancy course was uncomplicated. She states that she is doing well following delivery of the baby except that she has noticed some episodes of sadness and tearfulness. She is eating and sleeping normally and has no strange thoughts or thoughts of hurting herself or others. Physical examination is within normal limits for a patient who is status post cesarean delivery. Which of the following is the most likely diagnosis? A. Postpartum blues B. Postpartum depression C. Postpartum mania D. Postpartum psychosis E. Poststerilization depression

The correct answer is A. Postpartum blues is the term used to describe a common postpartum reaction that occurs in 50 to 70% of postpartum patients. It is characterized by tearfulness, restlessness, and anxiety. Symptoms typically start in the first few days postpartum and resolve within 2 weeks. Certain patients continue to have the symptoms for several weeks, however. Many symptoms may be seen in association with this disorder, including headache, backache, fatigue, forgetfulness, insomnia, weeping, depression, anxiety, and negative feelings toward the newborn infant. Interestingly, another component of the syndrome may be episodes of elation, and such mood lability can be especially distressing for the new mother. It is unclear what the etiology of these symptoms is. Certainly the postpartum period with a newborn can be stressful and life changing, which can certainly lead to mood changes and a number of emotional responses. Some researchers have argued that changes in hormone levels are at the root of the postpartum blues, but this has never been definitively proven. This patient does not have evidence of a true postpartum depression (e.g., insomnia, lack of appetite, or anhedonia) or postpartum psychosis (e.g., bizarre thoughts) and she does not have any thoughts of hurting herself or her infant. The most likely diagnosis is therefore postpartum blues and she should be given support and reassurance. The patient must also be cautioned, however, that if her symptoms do not resolve, or if they worsen, then she must call or return. Postpartum blues symptoms include mood swings, anxiety, sadness, irritability, crying, decreased concentration, and trouble sleeping. Postpartum depression symptoms are more severe and long-lasting and interfere with a woman's ability to care for the child. Postpartum psychosis may include symptoms such as confusion and disorientation, hallucinations and delusions, paranoia, and attempts to harm the child or herself.

A 19-year-old girl is brought to the emergency department by her friends after a weeklong history of "bizarre behavior." The friends say that the patient has been "talking a mile a minute" and has claimed to have special powers from God. The patient has not slept in the past 4 days yet has been "running all over the place and spending to the limit on her credit cards." On mental status examination, the patient is found to be distracted with pressured speech and flight of ideas. When asked about her mood, the patient replies that she has never felt this good before. She denies any suicidal or homicidal intent, ideation, or plan. She has never been depressed in her life. There is no history of substance abuse and urinary toxicology is negative. Which of the following is the most likely diagnosis? A. Bipolar disorder, type I B. Cyclothymic disorder C. Major depressive disorder, with atypical features D. Mood disorder not otherwise specified (NOS) E. Schizophrenia, chronic paranoid type

The correct answer is A. The patient is most likely experiencing a manic episode that does not appear to be in the context of an intoxication or medical condition. Note that despite the name "bipolar" only a manic episode is required to make this diagnosis, and the patient's lack of a history of depression does not change this diagnosis to a mood disorder NOS. Bipolar I disorder is a recurrent and/or chronic mental condition marked by alternating periods of mood elevation and depression (documentation of depression is not necessary for making this diagnosis). To meet these criteria, there must also be impaired functioning. With ongoing disease, depressive symptoms predominate with bouts of mania occurring intermittently. Mood stabilizers are the medications of choice. If bipolar disorder is misdiagnosed as major depression and the patient is placed on an antidepressant, a manic episode can ensue and modification in therapy will be needed. Lithium is the classic medical therapy for bipolar disorder in patients who are rapidly cycling and who are not pregnant.

A 42-year-old woman is admitted for outpatient elective cosmetic surgery. After the surgery, she develops acute shortness of breath, and pulmonary embolus is diagnosed. Two days later, an emergency psychiatric consultation is called, as the patient has developed a shaking tremor, is pulling out her IV lines, and appears to be watching snakes crawl around the floor of her room and ants crawling on her skin. Which of the following aspects of the patient's history would be most likely to point to the diagnosis? A. Alcohol use B. Depression C. Past LSD use D. Schizophrenia E. Traumatic childhood

The correct answer is A. The patient's hallucinations, agitation, and tremor point to the diagnosis of delirium tremens (DTs), which should always be considered when a patient does not have access to alcohol after admission to the hospital. Alcohol withdrawal states can present with tremors, nausea, sleep disturbance, auditory and visual hallucinations, seizures, and confusion. Delirium tremens can ensue with seizures, hallucinations, and confusion. Benzodiazepines are the cornerstone of treatment. Tremors, the first symptom (also known as "the shakes"), occur 4-12 hours after the last alcohol ingestion, and can progress to delirium tremens that can last 1-5 days.

A 30-year-old woman calls her new primary care provider to request a refill of her alprazolam (Xanax), which had been prescribed by her former health care provider for severe anxiety. Her new health care provider has only seen her once, about a month ago, after the patient made several superficial lacerations on her wrists during a fight with her boyfriend. At that time she had been referred for emergency psychiatric evaluation. On the telephone, she describes intense anxiety related to an unstable relationship with a new boyfriend, binges on alcohol over the last few days, alternating irritability, anger, and depression, and recurrent vague suicidal thoughts, without a plan or intent to harm herself. Contact with her former health care provider reveals that these feelings and behaviors were unchanged over the 5 years that he had seen her for routine health maintenance examinations and minor illnesses. Which of the following is the most likely underlying diagnosis? A. Borderline personality disorder B. Dependent personality disorder C. Histrionic personality disorder D. Narcissistic personality disorder E. Schizotypal personality disorder

The correct answer is A. The probable diagnosis is borderline personality disorder. This disorder is marked by a pervasive pattern of instability of interpersonal relationships, self-image, and affect, as well as marked impulsivity by early adulthood. Criteria for this diagnosis that are evident in this patient's history include: affective instability marked by severe anxiety, irritability, and depression, inappropriate and intense anger, impulsivity as made evident by cutting behavior and binges on alcohol, a pattern of unstable and intense relationships, and recurrent suicidal ideation and behaviors. The chronicity of these behaviors also supports this diagnosis. Borderline personality disorder patients have several features that are associated with this condition: paranoia, odd thinking, grandiosity, restricted range of emotions, anger and irritability, excessive emotionality and unstable mood, impulsive behaviors, and anxiety and tension.

A 21-year-old man drops out of college. On questioning, he explains that he is so afraid of having a panic attack that he is no longer willing to sit in class because he would not be able to leave. The same fear has also led him to stop going shopping or to the movies, because he is afraid of having a panic attack in the ticket line or theater. Which of the following is the most likely diagnosis? A. Agoraphobia B. Conversion disorder C. Obsessive-compulsive disorder D. Social phobia E. Somatization disorder

The correct answer is A. This patient has agoraphobia. Although this condition was originally defined as the fear of open spaces or of the marketplace, a more functional, modern definition is a fear of panic attacks in situations from which it would be difficult to gracefully remove oneself. As we have here, a patient's fear may involve multiple settings and may progress to the point of markedly hampering daily functioning. Panic attacks may or may not actually have been experienced in the past in the particular settings that are of concern to the patient. Some cases resolve spontaneously; others pursue a waxing and waning course. Behavioral therapy is used to encourage patients to modify their activities. Antidepressants are helpful in patients who have coexisting depression. Panic disorder with associated agoraphobia and no other comorbidity is treated with cognitive behavioral psychotherapy with or without SSRIs or SNRIs. SSRIs are regarded as the optimal class of medications because of their broad spectrum of activity and safety and tolerance. When treating panic disorder in this setting, patients may require high doses of these agents. Patients who have these panic attacks suffer from brief periods of intense physical and psychologic symptoms that initially occur unexpectedly. When panic disorder is associated with agoraphobia, the patient develops persistent anxiety being placed in situations in which escape may be difficult.

An 18-year-old woman comes to her health care provider along with her mother, who states that she doesn't know what to do with her daughter. Ever since the daughter started modeling, she became concerned about being fat. Over the past 6 months she has lost 25 pounds. The daughter states that she has a fear of gaining weight. She has not had her menstrual period for 5 months. She denies use of laxatives or diuretics. On examination she is thin and has fine hair all over her body. Her weight is about 20% below her expected body weight. Her blood pressure is 90/70 mm Hg and pulse 54/min and regular. Which of the following is the most appropriate next step in management? A. Admit her to the hospital to reestablish weight and correct metabolic abnormalities B. Refer the patient for psychodynamic psychotherapy C. Send the patient for evaluation for electroconvulsive treatment D. Start the patient on hormone therapy to induce her menstrual cycle E. Tell the mother not to worry and send them home

The correct answer is A. This patient has lost a significant amount of weight in a short period of time. As indicated by her vital signs and percentage of weight loss, she may need a full evaluation of her physical status to determine and correct metabolic abnormalities because her refusal to eat has placed her in a potentially life-endangering situation. Patients with anorexia nervosa have <85% ideal body weight, have a distorted body image, and engage in restricting or binging behavior. Weight restoration with appropriate refeeding techniques is essential for prevention of potential complications such as heart failure, fertility problems, and osteoporosis. Early intervention is needed to prevent long-term psychiatric and physiologic complications. This condition remains dangerous, as it has the highest premature mortality of any psychiatric condition.

A 31-year-old man is brought to the emergency department by a friend because he has been complaining that he is having auditory hallucinations and tremors, along with associated nausea and vomiting. He feels very anxious. On Mini-Mental Status Examination, he scores 22 of 30, and he appears to be obtunded. From which of the following substances is this patient most likely withdrawing? A. Alprazolam (Xanax) B. Caffeine C. Cocaine D. Heroin E. Nicotine

The correct answer is A. This patient has the symptoms of withdrawal from benzodiazepines, such as alprazolam. Symptoms include insomnia, tremor, GI distress, hallucinations, and anxiety. Withdrawal from benzodiazepines can also be accompanied by generalized seizures. Withdrawal from benzodiazepines is classically seen as hallucinations, seizures, tremors, and nausea in an obtunded patient. The presenting anxiety is an important clue to the etiology. As a rule, withdrawal from a substance produces the exact opposite of symptom complex that ingestion of that substance causes.

A 42-year-old man has just been informed that he has poorly differentiated small cell carcinoma of the lung. When asked whether he understands the serious nature of his illness, the patient proceeds to tell his health care provider how excited he is about renovating his home. Which of the following defense mechanisms is this patient exhibiting? A. Denial B. Displacement C. Projection D. Rationalization E. Reaction formation

The correct answer is A. This patient is in denial about his serious illness. By talking about something totally unrelated, he is trying to avoid facing or talking about the bad news he has just received. Displacement (choice B) involves the transferring of feelings to an inappropriate person, situation, or object (e.g., a man who has been yelled at by his boss takes out his anger on his wife). Projection (choice C) is the attribution of one's own traits to someone else (e.g., a philandering husband accuses his wife of having an affair). Rationalization (choice D) involves creating explanations for an action or thought, usually to avoid self-blame. Reaction formation (choice E) is the unconscious changing of a feeling or idea to its opposite (e.g., a man acts very friendly toward a coworker when in fact he is unconsciously jealous). Defense mechanisms are ways in which people deal with negativities. These can be consciously or unconsciously produced. Denial consists of simply not acknowledging the bad news that a person is being given. With time, patients can grow to accept their bad news, but this is something that the patient needs time in order to accept.

A 37-year-old nursing assistant is admitted to the general medicine inpatient unit for evaluation of new onset of seizures. She recently was at a family picnic and accidentally discovered that she was an adopted child, by way of a family member who assumed she had known the true story. Initially she was in absolute shock and was unable to discuss it with her parents. She loves and respects her parents deeply and was brought up in a very religious environment; her adopted father was a minister. About a month ago her parents came to visit for Thanksgiving and stayed at her house for a few days. One day while finishing lunch she started complaining of a headache and soon afterward had a tonic-clonic seizure. She was taken to the emergency department. The initial evaluation was negative and she was advised to follow up in the clinic. Since then she has had several similar attacks and has not been able to go back to work. Her parents have stayed around to help, but the seizures have continued and she is admitted to the hospital. Neurologic and medical workup is completely unremarkable and the psychiatric consult team is asked for an assessment. After 3 days, the patient seems much better and is extremely interested in talking about her symptoms. If this patient were to have another seizure, which of the following features will most likely be present? A. Alternating flailing limb movements B. Disorientation C. Incontinence D. Loss of body tone E. Retrograde amnesia

The correct answer is A. This patient is most likely experiencing psychogenic seizures characterized by the lack of organized bilateral clonic jerks. Patients usually have asynchronous, alternating limb movements with suggestive pelvic thrusting. These seizures typically do not cause urinary or bowel incontinence or postictal confusion. Psychogenic, nonepileptic seizures can present with limb shaking, variable loss of consciousness, eye closure, side to side head shaking, pelvic thrusting, prolonged convulsant activity, and lack of response to benzodiazepines. Video-EEG monitoring can be used to differentiate these psychogenic seizures from true epileptic seizures. Serum prolactin levels can also be used in this differentiation, with patients who have psychogenic seizures having normal prolactin levels and patients who have true epileptic seizures having elevated prolactin levels.

A 74-year-old woman with a long history of type 2 diabetes mellitus undergoes surgery for small bowel obstruction. After surgery, she develops acute renal failure. She refuses to undergo dialysis despite the advice of her health care provider who then calls for an immediate psychiatric consultation. The patient tells the mental health specialist that she has lived a long life and does not want to be kept alive by or attached to a machine, even if it means she will die. A mental status examination shows that she is not psychotic, that she is fully oriented and alert, and that she has no fluctuations of cognition or level of consciousness. The patient's family is insistent that she be dialyzed immediately. Which of the following is the most appropriate statement the health care provider could make? A. The patient is aware of the consequences of her decision and does not show signs of a major psychiatric illness. B. The patient is competent to decide on treatment, and her refusal to undergo dialysis must be respected. C. The patient is competent to decide on treatment, but her refusal can be overruled because of a medical emergency. D. The patient is operating in a suicidal manner and should be committed for treatment against her will. E. The patient is temporarily incompetent, so start her on dialysis.

The correct answer is A. This patient raises one of the most difficult legal and ethical problems in psychiatry. It is important to understand that competency, or lack of competency (choices B, C, and E), can be determined only by a legal authority, such as a court of law. The role of mental health professionals is solely advisory in determining competency. In this situation, only if the patient is suicidal by virtue of a major psychiatric illness, or if the patient were subject to an immediate medical emergency, could treatment be involuntarily administered. The mental health professional's role is to assess a person's mental status for evidence of cognitive impairment, as well as to ascertain that the patient has a thorough understanding of the consequences of treatment decisions that are made. This patient does not meet criteria for treatment against her will (choice D), which requires both a mental disorder and the threat of impending immediate harm to self or others. Competence is a legal, not a medical issue. Only courts can decide competence. The mental health provider or psychiatrist determines the patient's ability to understand the consequences of treatment decisions based on examinations to test memory, judgment, and understanding. Clear evidence of incompetence includes: Patient is suicidal or can be harmful to others Patient is grossly psychotic or dysfunctional Patient's physical or mental state prevents simple communication

An emergency department provider is attempting to evaluate a 32-year-old Caucasian man with a long history of schizophrenia, disorganized type. The patient has been taking haloperidol (Haldol) for the past 4 years without significant adverse side effects. In obtaining the history, the health care provider asks the patient to describe any auditory hallucinations he hears and the patient responds by saying, "killin, chillin, and thrillin." Which of the following most accurately describes this type of response? A. Clanging B. Flight of ideas C. Looseness of associations D. Neologism E. Thought blocking

The correct answer is A. When a patient associates words similar in sound but not in meaning, this is known as a clang association. In this case the words have no logical connections but sometimes rhyme or have similar phonetic phrasing. Several abnormal thought processes are associated with schizophrenia, such as loosening of association, clang associations, tangential thoughts, thought blocking, and thought broadcasting.

A 40-year-old woman is brought to the hospital after overdosing on alcohol and pills. In talking to a mental health provider, she denies any prior psychiatric problems but says that about a week ago her apartment burned down. She was trying to get a job before that happened, but when she realized that she had lost everything she had, and that moving in with her family would probably not be possible, she decided to take her own life. Which of the following is the most likely diagnosis? A. Acute stress disorder B. Adjustment disorder C. Antisocial personality disorder D. Brief psychotic disorder E. Major depressive disorder

The correct answer is B. Adjustment disorder is exemplified by a set of behavioral or emotional symptoms developing as a response to an identifiable stressor within 3 months after exposure to the stressor. The symptoms are excessive compared with what one would expect from the exposure, and they cause marked impairment in social functioning. Adjustment disorder involves maladaptive behavior that occurs within 3 months of the acute stressful episode in a patient's life. A patient may feel so hopeless and helpless following this life-changing event that the person does not feel that there are any other solutions except to try to commit suicide. The behavior that the patient exhibits is in excess to what a normal response to stress would have been.

A 64-year-old man with generalized abdominal pain and jaundice is told by his health care provider after an extensive workup that he has pancreatic adenocarcinoma. In the days following the diagnosis, the patient states to his doctors that, "I don't have cancer—all I need is a vitamin B12 shot and I will feel better fast." The patient is oriented at all times, scoring 29/30 on a Mini-Mental Status Examination, but refuses all chemotherapy medications and adamantly refuses to be seen by the hospital oncologist, because "he is a cancer doctor and I don't have cancer." Which of the following diagnoses would be most applicable in this case? A. Acute stress disorder B. Adjustment disorder C. Alzheimer dementia D. Delusional disorder E. Major depressive disorder

The correct answer is B. Adjustment disorder is the development of emotional or behavioral symptoms in response to an identifiable stressor occurring within 3 months of the onset of the stressor. These symptoms or behaviors are clinically significant as evidenced by marked distress in excess of what would be expected from exposure to the stressor, or by significant impairment in social or occupational functioning. In this case, the stressor is the diagnosis of pancreatic cancer, which impairs the patient's ability to function in the health care provider-patient relationship without denial. Situational adjustment disorder has a clearly identified precipitating event. This identifiable stressor has to be present within 3 months of the patient developing the adjustment disorder. The reaction to this stress is significantly more severe than would be expected and this causes impairment in function.

A 39-year-old woman with a history of hypomanic episodes in the past comes to her health care provider complaining of a depressed mood, lack of energy, feelings of hopelessness and guilt, and a decreased appetite for the past 5 weeks. She has never had similar symptoms in the past. Given this history, which of the following is the most likely diagnosis? A. Bipolar I disorder B. Bipolar II disorder C. Cyclothymic disorder D. Major depressive disorder E. Substance abuse

The correct answer is B. Bipolar II disorder is characterized by the presence or history of one or more depressive episodes and one or more hypomanic episodes. Hypomania is defined as a distinct period of persistently elevated, expansive, or irritable mood, lasting at least 4 days, but not severe enough to cause marked impairment in social or occupational functioning or to necessitate hospitalization. Bipolar II disorder is associated with hypomania and major depression symptoms that are associated with an impairment in functioning. Patients are primarily depressed and need to have at least one hypomanic episode to meet the criteria. Diagnosis is based on patient interviewing. Mood stabilizers are the most commonly used treatment strategy.

A 65-year-old woman comes to a primary care provider with a complaint of poor memory. Her husband died 6 months ago and she has recently decreased her church involvement and interest in her hobbies. She complains of feeling tired and worried, and has difficulty with attention and memory. She has a history of a major depressive episode approximately 10 years ago, which required hospitalization. She was treated with an antidepressant for 2 years until it was tapered and discontinued. She currently denies depressed mood. She states that her mother died of Alzheimer disease, and that she finds her memory loss particularly worrisome in light of this family history. Which of the following factors would be more likely if this patient has a pseudodementia of depression rather than a true dementia? A. The patient appears unconcerned B. The patient emphasizes disability related to memory loss C. The patient has more difficulty with recent memory than remote memory D. The patient tends to conceal the difficulty E. The patient tries hard to answer questions about orientation and registration

The correct answer is B. Distinguishing a pseudodementia of depression from a true dementia is an important task requiring examination of subtle aspects of the mental status examination. Although these individual factors may be used in the assessment of depression-related cognitive dysfunction, the entirety of the mental status examination must be considered when making diagnostic decisions. Treatment for the two conditions would be very different, and memory loss in this woman should not be automatically presumed to be related to a primary dementia. In pseudodementia of depression, the patient often tends to emphasize disability related to memory loss much more than patients experiencing true dementia. Pseudodementia is a condition that may mimic true dementia but these symptoms are present because the patient is not cognizant enough to have the insight that depression and not dementia is the underlying cause of these symptoms. Patients who have depression will have preserved visuospatial function and successful treatment of this condition with antidepressants will improve the mental status examination on these patients. Interpretation of mental status deficits should keep in mind that the depressed patient may not be trying hard to answer these questions correctly.

A 22-year-old woman comes to the mental health care professional's office after having been referred by her family physician. She reports no mental problems but tells the psychiatrist that she is concerned about hair loss. Her mother says that the patient has been grooming herself frequently and checking her image in the mirror. Her family physician has examined her several times; there is no evidence of hair loss. When the physician pulled on her hair, there was no loss of hair from the head. He has also ran multiple tests, but there is no evidence of any abnormality. The woman states that she is embarrassed and tries to avoid going out because everyone "will notice" and ask questions. She denies pulling her hair. Despite this problem, she appears well-adjusted and does a good job at work. Which of the following is the most likely diagnosis? A. Alopecia areata B. Body dysmorphic disorder C. Hypochondriasis D. Schizophrenia E. Trichotillomania

The correct answer is B. Body dysmorphic disorder, or dysmorphophobia, is the preoccupation with an imagined defect in appearance or excessive preoccupation with a slight anomaly present. It causes significant distress. Patients are often ashamed to present for treatment. They may frequently check their image and try to compensate for the imagined defect. Patients go on to develop preoccupation with their physical appearance. The disorder leads to social isolation secondary to imagined mockery. It can cause iatrogenic complications, like undergoing cosmetic surgery. Treatment typically focuses more on prevention of iatrogenesis, but it can also include selective serotonin reuptake inhibitors (SSRIs). Body dysmorphic disorder, or dysmorphophobia, is the preoccupation with an imagined defect in appearance or excessive preoccupation with a slight anomaly present. Remember that body dysmorphic disorder is not limited to the habitus, as such a disorder can include hair and nails. Treatment typically focuses more on prevention of iatrogenesis and cognitive therapy for the patient to gain insight into their abnormal preoccupation with supposed body defects.

Cognitive therapy for depression is based on the theory that the disorder is caused by which of the following? A. Anger toward others that is turned on the self B. Distorted negative beliefs and self-talk that reinforces those beliefs C. Fluctuating neurotransmitter levels D. Genetic predisposition E. Significant object loss in the first 3 years of life

The correct answer is B. Cognitive therapy is based on the theory that depressed persons have acquired negative cognitions about themselves, reinforcing these cognitions with self-talk that is often not verbalized. The goal of therapy is to identify the negative cognitions, question the validity of the beliefs, and substitute more positively reinforcing self-talk for the aversive self-criticism. Cognitive therapy for major depression has efficacy in managing this condition, and it can be used as stand-alone treatment or as an adjunct with medications. Cognitive therapy consists of having the patient eliminate negative thoughts by replacing them with positive thoughts to overcome the depression.

A 20-year-old woman suddenly develops the loss of voluntary movement of her right upper extremity. This occurred the morning that she moved back into her parents' home following a separation from her husband. The patient adamantly states that she is unable to move her arm. Her affect is constricted. She has full strength in all extremities, except her right upper extremity. Reflexes are symmetric and 2+ in all extremities, including the right upper extremity. Muscular tone is also symmetric in all extremities. Serum chemistries, vitamin B12/folate, thyroid function tests, and CBC are all within normal limits. MRI of the head and cervical spine are normal. Which of the following is the most likely diagnosis? A. Body dysmorphic disorder B. Conversion disorder C. Hypochondriasis D. Major depression disorder E. Malingering

The correct answer is B. Conversion disorder is one of the earliest described disorders in psychiatry. It involves the presentation of at least one neurologic symptom without an explainable etiology. The patient with conversion disorder is believed to be dispelling psychic conflicts by exchanging or "converting" them into neurologic symptoms. A common feature can be lack of affect or concern, despite what should be very troubling neurologic symptoms (paralysis, blindness), which is termed "la belle indifference." Conversion disorder symptoms are not consciously produced. Conversion disorder is a type of somatoform disorder in which patients develop somatic symptoms (typically at least one of the symptoms has to be neurologic in nature) that are not explained by other medical or psychiatric disorders. These symptoms classically occur after a psychologic stress, and the symptoms are unconsciously produced. Long-term management involves maintaining the same health care provider and encouraging high levels of emotional processing while acknowledging that the symptoms are real but are not disabling.

A 53-year-old veteran has a history of post-traumatic stress disorder (PTSD). He has been fairly stable after having gone through virtual reality treatment for his PTSD. He was recently diagnosed with hepatitis C and has started taking interferon. His wife is concerned that he has developed "the side effect of interferon therapy" that his gastroenterologist warned him about. Which of the following psychiatric symptoms is seen most commonly in patients who have hepatitis C treated with interferon? A. Anxiety B. Depression C. Flashbacks D. Hallucinations E. Mania

The correct answer is B. Depression is a side effect of interferon in patients who have hepatitis C. It is important to rule out the presence of suicidal ideation and monitor the symptoms carefully. Interferon, a common treatment for hepatitis, is known to cause depression in many patients. Patients should be carefully monitored for this complication and assessment for suicide ideation should also be performed.

A 24-year-old student is brought to the emergency department because of a sudden change in behavior. His friends report that he has been acting "weird" and confused and has been talking about "flying above the floor." He laughs without a reason and then burst into tears. He had been at a party the night before and may have used some drugs. He appears hot and has uncoordinated movements. He has horizontal nystagmus, ataxia, and muscular rigidity. He becomes very agitated and combative during the examination. He is placed in a nonstimulating environment. Which of the following agents should be administered at this time to treat his agitation? A. Chlorpromazine (Thorazine) B. Diazepam (Valium) C. Flumazenil (Romazicon) D. Naltrexone (Revia) E. Trihexyphenidyl (Artane)

The correct answer is B. Diazepam (Valium) is used for treatment of agitation related to phencyclidine (PCP) intoxication. Benzodiazepines may be effective in treating violent behavior. Cranberry juice or ascorbic acid may be useful in acidifying the urine and speeding up the elimination of the drug. Physical symptoms of phencyclidine (PCP) intoxication include nystagmus, flushing, diaphoresis, and hyperacusis. Behavioral changes during acute intoxication include distortions of body image, disorganized thinking, and feelings of estrangement. Initially patients may have anxiety, paranoia, delusions, and hallucinations. Treatment of acute intoxication includes medical support for these patients who may have fever and hypotension. Patients may be helped by acidification of urine or use of gastric lavage if oral ingestion is recent.

An infant born to an alcoholic mother had microcephaly and cardiac abnormalities and died despite resuscitative efforts. During which of the following periods of pregnancy is alcohol most teratogenic? A. First 2 weeks B. 3rd through 9th weeks C. 9th through 12th weeks D. 12th through 16th weeks E. 16th through 20th weeks

The correct answer is B. Embryonic tissue is most susceptible to teratogens during the 3rd through 9th weeks of pregnancy. This is when organogenesis, as well as most major congenital anomalies, occurs. Because brain development occurs throughout pregnancy, however, it is wisest for a pregnant woman to avoid alcohol for all 9 months of gestation. Embryonic tissue is most susceptible for teratogenicity between weeks 3 through 9, which makes alcohol ingestion at this time a cause for concern for the development of fetal-alcohol syndrome. This is especially concerning because this time frame would include a time period prior to a woman being aware that she was pregnant.

A 32-year-old woman with a history of panic disorder is brought to the emergency clinic by her husband after he found her heavily sedated and minimally responsive upon his arrival home from work. The husband informs the health care provider that his wife had become increasingly depressed over the past few weeks since the death of her mother. He states that her psychiatrist had recently increased her doses of alprazolam (Xanax) and sertraline (Zoloft) because of increasing panic attacks along with worsening depression. Her temperature is 37º C (98.6º F), blood pressure 105/65 mm Hg, pulse 65/min, and respirations 12/min. She is heavily sedated and only able to utter a few phrases, with slurred speech. She did state that she took a diazepam (Valium) and several extra alprazolam (Xanax) because of panic attacks. During the physical examination her respiratory rate drops to 8/min. Which of the following medications would be the most appropriate to administer at this time? A. Benztropine (Cogentin) B. Flumazenil (Romazicon) C. Haloperidol (Haldol) D. Lorazepam (Ativan) E. Naloxone (Narcan)

The correct answer is B. Flumazenil is a benzodiazepine receptor antagonist, which reverses the psychophysiologic effects of benzodiazepine medications, such as alprazolam, diazepam, and lorazepam. After IV administration, flumazenil has a half-life of 7-15 minutes. For the initial management of a known or suspected benzodiazepine overdose, the recommended initial dose of flumazenil is 0.2 mg administered IV over 30 seconds. If the desired level of consciousness is not obtained after waiting 30 seconds, a further dose of 0.3 mg can be administered over 30 seconds. Additional doses can be administered up to a cumulative dose of 3.0 mg. A secure airway and IV access should be established before administration of the drug. Flumazenil works as an antagonist to benzodiazepine overdose. Because of its short half-life, it may need to be used every minute for up to 5 doses to obtain a response. It can then be repeated every 20 minutes if the patient develops sedation again if a long-acting benzodiazepine was ingested.

A 32-year-old woman comes to a mental health provider because she "worries too much." She admits that over the past 7 months she has experienced extreme fatigue, muscle tension, and irritability. She has difficulties falling asleep, partly because of worrisome thoughts about her husband and children. She keeps worrying that something bad is going to happen to them. In the past month, she has had episodes of shortness of breath, dizziness, and restlessness, and has been unable to go to work or do anything at home. Her physical examination and laboratory tests, as well as her electrocardiogram, are unremarkable. Which of the following is the most likely diagnosis? A. Avoidant personality disorder B. Generalized anxiety disorder C. Hypochondriasis D. Obsessive-compulsive disorder E. Panic disorder

The correct answer is B. Generalized anxiety disorder is defined as unrealistic worry about life events for a period longer than 6 months, during which time a person is worried most days. It also includes at least 6 symptoms of the following types: easy fatigability, difficulties falling asleep, restlessness, difficulties concentrating, irritability, and muscle tension. The symptoms are not caused by other psychiatric or medical conditions and they cause significant impairment in everyday functioning. Generalized anxiety disorder is a mental condition in which the patient experiences chronic, excessive worry for at least 6 months that causes impairment in functioning. To meet the criteria, patients should have restlessness or nervousness, easy fatigability, poor concentration, irritability, muscle tension, or sleep disturbance. Treatment involves cognitive behavioral therapy. For patients who need additional medication, SSRIs are considered the medication of choice.

A 38-year-old man is brought to the health care provider by his mother. The mother states that the patient has begun to exhibit "bizarre movements that make him look like he's impersonating a snake." He has also become increasingly paranoid and irritable over the last 2 years. He has few friends and has never moved out of the family house in which he grew up. The patient's father died in an assisted living facility at the age of 45 after having a similar initial presentation. Which of the following is the most likely diagnosis? A. Amnestic disorder, not otherwise specified B. Huntington disease C. Parkinson dementia D. Senile dementia, Alzheimer type E. Vascular dementia

The correct answer is B. Huntington disease is an autosomal-dominant disorder that involves degeneration of neurons in the basal ganglia, producing choreoathetoid movements and psychiatric symptoms that may include depression, psychosis, personality changes, and dementia. Huntington disease is an autosomal-dominant inherited disorder that causes progressive neurodegenerative deterioration. It typically presents in middle age with chorea-like movements, poor coordination, cognitive decline, and personality changes. Lifespan is affected, with death typically ensuing within 20 years of initial symptoms.

A 78-year-old woman is complaining to the health care provider in her nursing home about her new problems. She reports that her old boyfriend from 50 years ago called her and is now harassing her by controlling her blood pressure, movements, and thoughts. She is convinced that he was initially doing this through the phone by sending special signals with a device that the CIA uses, but now he is able to control her through his thoughts. She stated that she couldn't tell the police because they probably would not believe her. This patient most likely has which of the following thought disorders? A. Clang associations B. Ideas of influence C. Ideas of reference D. Noesis E. Obsessions

The correct answer is B. Ideas of influence constitute a type of delusion in which a person believes that he or she is being controlled by another person or external force. Know the difference between ideas of influence and ideas of reference. With ideas of influence, a person believes that he is being controlled by another person or external force. With ideas of reference, a person falsely believes that others (including people on TV or radio) are talking about him.

A couple visits their primary care health care provider because of sleep difficulties. The wife reports that she cannot fall asleep at night because of her obese husband's loud snoring. He complains of feeling drowsy during the day and falling asleep while driving home from work. Which of the following conditions does the husband most likely have? A. Central sleep apnea B. Obstructive sleep apnea C. Narcolepsy D. Pavor nocturnus E. Somnambulism

The correct answer is B. In an obese patient, loud snoring, daytime drowsiness, and falling asleep while driving are all classic clues to the diagnosis of obstructive sleep apnea. Sleep apnea is defined as the cessation of airflow for at least 10 seconds. The risk factors for sleep apnea include obesity, abnormal jaw placement, oropharyngeal narrowing, and macroglossia. Clinical features of obstructive sleep apnea include loud snoring, gasping during sleep, unrefreshing sleep, and excessive daytime sleepiness. Polysomnography and sleep studies are used in making the diagnosis. Patients with obstructive sleep apnea respond well to continuous positive airway pressure (CPAP) that is applied when going to bed.

A 60-year-old man has had difficulty falling asleep for the past several weeks and asks his health care provider for advice. The patient has no underlying psychopathology. Which of the following is the most appropriate nonpharmaceutical treatment for this patient? A. Increasing fluid intake during the 3 hours prior to sleep B. Retiring for sleep at the same time each day C. "Sleeping in" if the time of retiring has been delayed D. Staying in bed until sleep is attained E. Strenuous exercise 1 hour before retiring

The correct answer is B. In the treatment of "normal" insomnia (in which the patient has no underlying psychopathology), it is important to establish a routine that does not vary. The person therefore should retire and rise at the same time each day. Sleep hygiene techniques include retiring at the same time every night, avoiding strenuous activities and caffeine several hours before bedtime, making lists of things that the patient is concerned about prior to retiring (allowing the person to put these concerns "to rest"), and limiting naps during the day so that the body is physically tired to encourage sleep.

A 9-year-old boy is brought to the clinic with a 3-month history of multiple episodes of sudden awakening at night. His mother states that when he wakes up suddenly, he screams, "Go! Get away! Go!" and does not respond to the parents. His eyes are wide open, and he sweats heavily and looks scared. The parents have had to struggle to awaken him. After the episodes, he has no memory of what happened. The boy is well adjusted to his home and school life. Which of the following is the most likely diagnosis? A. Confusional arousals B. Night terrors C. Nightmares D. Obstructive sleep apnea E. Panic disorder

The correct answer is B. Night terrors are a form of parasomnias, which refer to unusual behaviors that occur in the context of sleep, in specific sleep stages, or in connection with arousal from sleep. Night terrors are most common in children age 4-12 and typically occur within the first several hours of sleep. The child suddenly cries out, sitting up in bed with a terrified look, crying inconsolably, perhaps thrashing about, and exhibiting evidence of increased autonomic arousal with enlarged pupils, tachycardia, rapid breathing, and sweating. Episodes typically last only a few minutes, and the child then returns to sleep, with no memory of the events the next morning. Night terrors are believed to be disorders of arousal from non-REM sleep (stages 3 and 4) in which motor behavior occurs but conscious awareness and memory of the action are not present. They are more likely to take place during periods of illness, stress, or sleep deprivation, but they can happen without any obvious associated stress. Those subject to night terrors or somnambulistic events should avoid sleep deprivation, which can increase the likelihood of their occurrence. Most children who have infrequent night terrors grow out of them with maturity and usually require no specific treatment. Clinicians should thoroughly explain the phenomenon to the parents and reassure them that the child is well. The expected eventual remission of the problem should be emphasized. Parents are encouraged not to awaken the child, but to allow the episode to run its course. If the child is not awakened, he or she will return to normal sleep at the end of the episode. If the child thrashes about wildly during the episode, the parents should provide protection from injury at that time. Confusional arousals (choice A) start gradually (unlike a full sleep terror in older individuals, which begins precipitously), with moaning progressing to crying, sitting, and thrashing. The children are difficult to arouse and do not respond to comforting, but when allowed to return to sleep, they do not typically remember the event the next morning. Nightmares (choice C) are frightening dreams that awaken the child from REM sleep. The child becomes fully awake and is scared. He or she usually can recall details of the dream. Obstructive sleep apnea (choice D) manifests as apneic episodes during sleep in which the patient awakens suddenly. Affected patients usually are obese. Panic disorder (choice E) is characterized by recurrent panic attacks which at the outset may occur spontaneously, and over time, may develop in a number of agoraphobic situations. The patient may experience a sense of terror or fear associated with a panic attack, including concerns about dying or losing control. Parasomnias are common occurrences in children. Night terrors involve screaming, yelling, incoherent moaning, diaphoresis, and visible anxiety which lasts 10-20 minutes during which time the child is inconsolable. After the episode, the child has no recollection of the event in the morning. These episodes occur during the first half of the night. Nightmares involve vivid, frightening dreams that the child awakens from and has full memory of the episode.

A 27-year-old female swimmer not only feels insecure about her athletic abilities but also criticizes her teammates' techniques. Which of the following ego defense mechanisms is she displaying? A. Displacement B. Projection C. Reaction formation D. Repression E. Sublimation

The correct answer is B. Projection involves attributing one's own traits, feelings, and attitudes to someone else. This swimmer's harsh criticism of her teammates' abilities is a reflection of her personal feeling of incompetence. (Doubts about her own ability are translated into doubts about her teammates' aptitude.) Patients experiencing acute stress reaction may develop defense mechanisms as one way to deal with stress. Patients may project their own securities on others, which may help them deal with their stress. When people attribute their own thoughts and feelings onto others in a critical way, projection is being used as a defense mechanism.

A 22-year-old student is brought to the emergency department by friends because of odd behavior and insomnia for 3 nights. He recently began talking about an ex-girlfriend who he believes planted listening devices in his apartment to report secret conversations to the police. He has been irritable and accused his roommate of putting poison in the tap water to make him forgetful and unable to focus on his studies. He has intermittently used "speed," "ecstasy," and other stimulants for the last few years. In the emergency room the patient is very irritable and points to "hidden microcameras." He seems to listen attentively but constantly turns around, looking to see where the voices are coming from. He has no prior psychiatric history, and his family history is not contributory. Physical and neurologic examinations are unremarkable. A diagnosis of schizophrenia or psychosis-associated with chronic abuse of amphetamines is being considered. Which of the following symptoms is more consistent with a diagnosis of schizophrenia? A. Apathy and withdrawal B. Disordered thought associations and affective flattening C. Hallucinations D. Paranoia E. Stereotyped movements

The correct answer is B. Schizophrenia is characterized by disordered thought, paranoid thinking, and odd behavior, and the only distinction from stimulant-induced psychosis is the presence of a formal thought disorder and affective flattening. Thought disorder is characterized by loose associations that render speech and thoughts as vague, unfocused, or inexact. Affective flattening is defined as absence or severely restricted subjective feelings. Patients who have schizophrenia will often have a + family history along with an increased risk for the use of drugs such as amphetamines and other stimulants that may exacerbate the psychosis. Patients who have schizophrenia often have auditory hallucinations, delusions, avolition, anhedonia, asocial behavior (not wanting to engage with others), and alogia (decrease in speech). Positive symptoms include tangentiality and looseness of association. Substance abusers may present with signs and symptoms consistent with schizophrenia, but they typically do not have thought disorders or flattening affects that are classically seen with schizophrenia.

An infant is brought to the clinic for a routine health visit and vaccinations. She is the product of an uncomplicated pregnancy and has been meeting all developmental milestones. She is feeding well, and her mother reports that the infant seems to be growing well also. On physical examination the infant is afebrile with stable vital signs. She can lift her head to 90 degrees in the prone position and her eyes follow past the midline. She laughs, regards her own hand, and has slight awareness of her mother. Which of the following is the most likely age of this infant? A. 2 months B. 4 months C. 6 months D. 12 months E. 18 months

The correct answer is B. The ability to lift the head to 90 degrees, eyes crossing the midline, laughing, and slight awareness of the caregiver are characteristic developmental milestones of a 4-month-old infant. A few characteristic developmental milestones at age 4 months are lifting the head to 90 degrees in the prone position, following past the midline with the eyes, laughing, and having slight awareness of the mother.

A 27-year-old man believes that he lives on a planet circling the star Alpha Centauri. His speech is rambling, he giggles frequently at inappropriate times, and he does not make sense when talking with others. He exhibits purposeful behaviors that do not accomplish an end and often "hears voices." Which of the following is the most likely diagnosis? A. Catatonic schizophrenia B. Disorganized schizophrenia C. Paranoid schizophrenia D. Residual schizophrenia E. Undifferentiated schizophrenia

The correct answer is B. The disorganized type of schizophrenia is characterized by disorganized speech, disorganized behavior, and an inappropriate affect. This patient's rambling speech and inappropriate giggling illustrate this condition. Disorganized schizophrenia patients have psychosis with delusions or hallucinations, disorganized speech and inappropriate giggling. Patients may also have bizarre behavior or bizarre delusions and may have an inappropriate affect.

A 31-year-old woman with schizophrenia is brought to the emergency department by the police because she was found in a park, actively hallucinating and talking to herself. She is cooperative but irritable. She denies command hallucinations and is not violent or suicidal. She admits that she ran out of her quetiapine (Seroquel) several days ago and was unable to get in touch with her case manager. She was living in a personal care home for 3 months and was stable on her medication, but she left there yesterday and didn't go back. Which of the following is the most appropriate next step in management? A. Admit her to the psychiatry inpatient service for further treatment B. Contact her case manager to ensure support and follow-up C. Convince her to take a haloperidol decanoate shot (Haldol) and stay "medicated" for a month D. Give her a new prescription and discharge her to the personal care home E. Send her back to the personal care home and let the owner handle her

The correct answer is B. The most appropriate step is to contact the case manager to verify the information and to ensure that she gets the care and the medication she needs. She may be medicated in the emergency department and then sent to the personal care home with the case manager or the personal care homeowner. Schizophrenia is a chronic psychiatric condition that responds to psychotropic medication. Compliance with medication can be difficult and patients often regress if the medication is stopped. Case-worker support and follow-up can help patients continue to be functional in society.

A 34-year-old woman comes to the health care provider because she feels like she is an "absolute loser" in everything she does. She recently started a new job in accounting but made a miscalculation last week and had a terrible conflict with her boss. She felt like an idiot at the time and has continued to feel that whatever she does is not adequate. As an example of her inadequacy, she continues to provide a list of all her mistakes and failures. She ends the interview by stating that she must be the unluckiest person in the world, because it seems that all these problems happen only to her. She has been depressed in the past but she does not know if medication can change her life events and make her feel differently about herself. Which of the following is the most appropriate management? A. Address patient's early attachment issues and work on possible developmental deficits B. Challenge the patient's negative beliefs C. Explore interpersonal relationships by working on role disputes D. Suggest participant modeling technique to help with the problems E. Suggest that self-employment will help her avoid being criticized

The correct answer is B. The patient has many cognitive distortions and negative beliefs about herself. The most appropriate and efficient technique to use would thus be to apply cognitive therapy and challenge the negative beliefs. Cognitive therapy can be utilized in the management of several psychiatric conditions such as major depression, anxiety disorders, phobias, and distortions of self with low self-esteem. Cognitive therapy can be used in concert with medical therapy and can be used to help the patienst to gain insight into their psychiatric issues so that thought redirection can be made.

A 35-year-old man is brought to the emergency clinic by his mother because of an episode of slurred speech associated with the uncomfortable sensation that his tongue is thick and curling up. The episode started suddenly 30 minutes ago. The patient is noted to be holding on to his tongue with his thumb and forefinger. When asked about this, the patient responds with dysarthria, saying that his medication has caused this once before and that he needs a shot to make it go away. His mother reports that the patient has had schizophrenia for 10 years and consistently takes 2 medications prescribed by his psychiatrist. Several days ago he ran out of one of his medications, but has continued to take the other one. Administration of which of the following should be included in the initial step in the management of this patient? A. Alprazolam (Xanax) B. Benztropine (Cogentin) C. Haloperidol (Haldol) D. Lorazepam (Ativan) E. Olanzapine (Zyprexa)

The correct answer is B. The patient is experiencing an acute dystonic reaction, which is a form of extrapyramidal side effect (EPS) associated with antipsychotic medications. These side effects are related to antagonism of dopamine receptors in the nigrostriatal pathway. The patient is likely taking a conventional antipsychotic agent plus a prophylactic anticholinergic agent (such as benztropine, diphenhydramine [Benadryl], or trihexyphenidyl [Artane]). Upon stopping the anticholinergic, the dystonic reaction was more likely to occur. The appropriate initial management of this patient would include immediate IM administration of an anticholinergic agent, such as 2 mg of benztropine or 50 mg of diphenhydramine. Acute dystonic reactions sometimes occur as a result of short-term use of antipsychotics or metochlopramide (Reglan) if taken for at least 12 weeks. In addition to stopping the offending medication, other agents can be used in the treatment or prevention of this painful condition, such as benztropine, diphenhydramine, or trihexyphenidyl. These agents work by antagonizing the acetylcholine and/or histamine receptors.

A 27-year-old office manager with a history of generalized anxiety has had multiple recent visits to the health care provider for a several-month history of fatigue. Evaluation thus far has revealed a normal physical examination as well as screening laboratories that are within normal limits. The serum thyroid stimulating hormone level is 2.3 µU/mL (normal 0.4-4.0 µU/mL). Inquiring about specific symptoms of depression on the last visit yielded the diagnosis of depression, and treatment options were discussed with her. On this visit she expresses frustration that a medical etiology of her fatigue has not been identified and she demands to see the clinic notes from her last several visits. Which of the following would be the most appropriate response the health care provider could make? A. Agree to show her the records only after she undergoes treatment for her depression B. Attempt to arrange to review her clinic records with her as soon as possible C. Discuss her case with a lawyer prior to showing her the records D. Explain to her that health care provider-patient confidentiality prohibits her from seeing her medical record E. Immediately make copies of her medical record for her to review at her convenience

The correct answer is B. The patient's medical record is technically her property, and as such, she may review it at any time. Much of the documentation in the chart may be unclear to patients, however, and may be potentially misconstrued. For this reason, it is always best to first offer to review the patient's chart with them. The information contained on the medical record truly belongs to the patient. The health care provider or health care facility owns the medical record only physically, but the information contained in it is technically the patient's property, and as such, he or she has the right to access the contents at any time. Much of the documentation in the chart may be unclear to patients, however, and may be potentially misconstrued. It is therefore always best to first offer to review the patient's chart with them.

The father of a 4-year-old boy reports that a friend thinks that his son is developmentally delayed and needs therapy. Upon evaluation, the boy copies a cross and a square, hops and walks downstairs, knows his colors, and tells stories. He also helps with brushing his teeth and plays with other children in group play. Physical examination yields a normal neurologic examination, and during the examination the child asks, "Can I go play now?" Which of the following is the most appropriate next step in management? A. Draw blood for a complete blood count and electrolytes in search for a cause of his delay B. Reassure the father that his son has normal development C. Refer the boy to a neurologist to evaluate his delays D. Refer the boy to a physical therapist to help with his motor delay E. Refer the boy to a speech therapist to aid with his speech delay

The correct answer is B. This boy has achieved the milestones appropriate for a 4-year-old child. The major motor milestones include hopping, galloping, and alternating feet while walking downstairs. Fine motor milestones include copying a cross and a square and drawing a man with 2 to 4 parts besides the head. The patient should also know colors, be able to tell stories, and speak in full, understandable sentences. In addition, he should engage in group play (as opposed to the parallel play in which a 2-year-old is able to participate) and brush his teeth. This patient has achieved those milestones and therefore you should reassure this father that his child is developing well and requires no further evaluation. Growth and development milestones are assessed in patients to identify issues so that intervention can be made to positively affect the child's development. The American Academy of Pediatrics recommends development and behavior screening in all children and adolescents.

After a minor but distressing automobile accident, a patient is unable to move one leg. Careful physical examination demonstrates no obvious injury that might have caused the paralysis. His reflexes are intact. A CT scan of the spine demonstrates no back injury. The patient is reassured, and the paralysis resolves over a 2-week period. Which of the following is the most likely diagnosis? A. Body dysmorphic disorder B. Conversion disorder C. Munchausen syndrome D. Pain disorder E. Somatization disorder

The correct answer is B. This case illustrates conversion disorder. In this condition, physical symptoms are caused by psychologic conflict. The symptoms develop unconsciously and are, by definition, limited to those that mimic a neurologic disorder, such as impaired coordination, weakness, paralysis, loss of sensation, blindness, deafness, or inability to speak. The onset is usually abrupt and linked to a stressful event. In most patients, symptoms improve within 2 weeks, although some patients will have persistent or recurrent problems. Patients who have conversion disorder have unconscious production of physical symptoms that typically are precipitated by a psychologic stress. Patients' symptoms tend to resolve with reassurance, whereas patients who have hypochondriasis are typically not improved with reassurance and these patients tend to seek confirmation of the disease they think they have.

A 31-year-old woman has a 7-year history of binging and purging. She also exercises excessively, often for 4 hours, 6 days a week. She has not sought medical attention for this problem in the past, but for the past couple of months she has had a depressed mood, a lack of energy, poor sleep, and decreased ability to concentrate. She comes to the health care provider now complaining of these new symptoms and would like treatment. Which of the following medications would be the most appropriate to initiate? A. Bupropion (Wellbutrin) B. Fluoxetine (Prozac) C. Haloperidol (Haldol) D. Lithium carbonate (Lithobid) E. Valproic acid (Depakene)

The correct answer is B. This patient appears to have symptoms of major depression in the context of bulimia. She should therefore be treated with an antidepressant medication, and a selective serotonin reuptake inhibitor (SSRI), such as fluoxetine, would be most appropriate. Bulimia nervosa treatment involves cognitive behavioral therapy, interpersonal therapy, nutritional counseling and meal support, and medications such as SSRIs and SNRIs. Fluoxetine is considered a first-line medication option in this disorder, but other SSRIs can also be used.

A 26-year-old man with schizophrenia comes to the emergency department with a 2-hour history of involuntary contractions of the muscles in his neck. He states that he was watching television and "all of a sudden I turned my head and my neck locked." He began taking a high-potency antipsychotic agent 3 days earlier. Examination shows no abnormalities except torticollis. Which of the following is the most appropriate pharmacotherapy? A. Amantadine (Symmetrel) B. Benztropine (Cogentin) C. Bromocriptine (Parlodel) D. Clonidine (Catapres) E. Propranolol (Inderal)

The correct answer is B. This patient has acute dystonia. Dystonia is characterized by involuntary muscle spasms, which in this patient are caused by a high potency antipsychotic agent. Dystonia is most common in young men and often begins within days of starting the drug therapy. It usually involves the muscles of the head and neck, leading to torticollis and blepharospasm. It can produce a life-threatening laryngospasm requiring intubation. The treatment of acute dystonia is with anticholinergic medications, such as benztropine (Cogentin) or diphenhydramine (Benadryl). Benztropine (Cogentin) is used to treat extrapyramidal side effects and acute dystonic reactions that are associated with the use of antipsychotic agents. Its mechanism of action is antagonization of acetylcholine and histamine receptors.

A 34-year-old man comes to his mental health provider after missing his last appointment 2 months ago. The patient has a history of bipolar I disorder, for which he takes lithium carbonate. He has not taken his medications for the past 2 months. Upon presentation he is very loud and talkative, with pressured speech and racing thoughts. He states that he has not slept for the past 4 days because he has been working to establish a catering service, for which he has been buying many kitchen utensils, baking supplies, and recipe books. He has also been "sampling wines from around the world" so that he can become a wine expert for his new business. In assessing the patient's affect and thoughts, which of the following mental status examination findings is most likely present? A. Blunted affect B. Inflated self-esteem and grandiosity C. Inappropriate and excessive guilt D. Nihilistic delusions E. Repetitive handwashing and counting

The correct answer is B. This patient is most likely experiencing a manic episode. He has a history of bipolar I disorder, which is characterized by single or recurrent manic episodes, often alternating with major depressive episodes. Symptoms of a manic episode, lasting for at least 1 week, include: persistently elevated, expansive, or irritable mood, inflated self-esteem or grandiosity, decreased need for sleep, hyperverbal and pressured speech, flight of ideas, distractibility, increase in goal-directed activity, and excessive involvement in pleasurable activities. Acute mania, a component of bipolar I, may be associated with inflated self-esteem or grandiosity, decreased need for sleep, being more talkative than usual, flight of ideas, distractability, increased goal-directed therapy, excessive involvement in pleasurable activities (without concern for consequences), and functional impairment. When acute rapid cycling occurs, oral mood stabilizers or atypical antipsychotics (divalproex [Depakote], carbamazepine [Tegretol], risperidone [Risperdal], olanzapine [Zyprexa], or oraripiprazole [Abilify]) can also be used.

A 51-year-old man comes to the emergency department several days after being discharged from the hospital where he had an extensive workup for abdominal pain that was negative. He now complains of similar abdominal pain and when the health care provider tries to reassure him that his physical findings and studies are normal, he refuses to leave, accusing the team of "not doing enough." He insists on calling the surgeon. Instead, a psychiatry consult is called. The psychiatrists describe the patient's mental status as normal. There is no evidence of suicidal or homicidal ideation. The interview with the patient reveals, however, that he lost his wife after 24 years of marriage in a car accident 6 weeks ago. Which of the following is the most appropriate initial step in management? A. Admit the patient to psychiatry for further evaluation B. Explore the patient's feelings about the recent loss C. Prescribe an antidepressant D. Prescribe benzodiazepines on an as-needed basis E. Reassure the patient that his pain is most likely stress-related

The correct answer is B. This patient is obviously having a hard time, struggling with feelings of loss and grief. It is easier and socially more acceptable to have a somatic disease, such as abdominal pain. The patient should be encouraged to process his feelings of pain and anger about the loss. Grief and poor adjustment reaction are expected findings after the loss of a loved one. The patient may have physical manifestations caused by a somatoform disorder not otherwise specified because his symptoms have been present <6 months. Major depressive disorder can have physical manifestations as part of its condition. Patients who have normal grief reaction may have transient depression. Symptoms consistent with depression include inappropriate guilt regarding the death of a loved one, persistent thoughts of death (survivor's grief), preoccupation with worthlessness, marked psychomotor retardation, prolonged functional impairment, and hallucinations. Patients who have unremitting symptoms >4-6 months should be evaluated for major depression. Somatoform disorder may be developing because the patient is developing physical symptoms as a result of his emotional stress. Somatoform disorder is a diagnosis of exclusion because physical disease rather than emotional disease may be the cause of the patient's presentation.

A 17-year-old boy is brought to the hospital by paramedics because of possible antifreeze ingestion. The father reports that his son has recently been drinking alcohol excessively. Over the past few days, the son had seemed very depressed and had been caught sneaking alcohol into the house on 2 occasions. The father tried confining the son to his bedroom, but later found him face down on the garage floor near a bottle of antifreeze. Which of the following is the most appropriate initial step in establishing the diagnosis of suspected antifreeze ingestion? A. Evaluate for hyperemia of his optic discs B. Evaluate his urine under a Woods lamp C. Examine the patient's breath for a fruity odor D. Obtain an electrocardiogram E. Request a serum osmolarity measurement

The correct answer is B. To rapidly assess for the possibility of antifreeze ingestion, the health care provider can evaluate the patient's urine under a Woods lamp for fluorescence. Manufacturers of ethylene glycol-containing antifreezes typically add fluorescein to the mix, which will fluoresce under a Woods lamp. Calcium oxalate crystals may also be visible on urinalysis as a result of antifreeze ingestion. Ethylene glycol-containing antifreezes usually contain fluorescein. Examination of patient's urine under Woods lamp will fluoresce, which suggests the possible ingestion of antifreeze. Urinalysis may also show calcium oxalate crystals that can form from the ingestion of ethylene glycol. Treatment of this toxic ingestion is with fomepizole (Antizol) given intravenously.

A 33-year-old woman with bipolar disorder and history of alcohol and drug abuse is brought to the emergency department after being found at home. She was found unresponsive to voice and touch, with 16 empty packets of medication next to her, each containing 50 mg of diphenhydramine. There was also an empty bottle of acetaminophen and a half-empty whiskey bottle near her bed. A suicide note was present on a nearby table. In the field, her temperature was 36º C (96.8º F), blood pressure 90/55 mm Hg, pulse 140/min and regular, and respirations 8/min. Additional data obtained in the field included a finger stick glucose of 20 mg/dL and an oxygen saturation of 87% on room air. Of the data available in the field, which of the following findings is most immediately threatening to her outcome? A. Blood pressure of 90/55 mm Hg B. Finger stick glucose of 20 mg/dL C. Oxygen saturation of 87% on room air D. Pulse of 140/min E. Respirations of 8/min

The correct answer is B. When blood glucose level drops ≤25 mg/dL, as in this case, cerebral glucose reserves are quickly depleted. In response, protein and lipid components of neurons are metabolized, which can lead to irreversible brain damage. Prompt treatment with intravenous glucose is therefore critical in this patient's initial management. Patients who have significant hypoglycemia can have serious side effects. If there is not enough glucose in the serum, seizures can ensue; these should resolve with the correction of hypoglycemia. Lack of glucose availability in the brain can also cause coma, which can also resolve with correction of the hypoglycemia. Permanent complications may occur because of hypoglycemia and this may involve local neurologic deficits and memory loss.

A 4-year-old child is brought to the pediatric clinic for a health maintenance visit. He is healthy and up to date on his vaccinations. His mother is concerned about his development because she has been comparing him to other children in his preschool. Which of the following skills would be expected of a 4-year-old? A. Building a 10-cube staircase B. Drawing a person with 6 parts C. Drawing a square D. Drawing a triangle E. Repeating 5 digits

The correct answer is C. A 4-year-old child is able to draw a 4-sided figure (i.e., a square), count to 4, identify 4 colors, say a 4- to 5-word sentence, and draw a picture of a person with at least 4 parts. Remember this easily as 4-year-olds do things in 4s. Developmental milestones reached by a 4-year-old child include: drawing a 4-sided figure, counting to 4, identifying 4 colors, saying a 4- to 5-word sentence, and drawing a picture of a person with at least 4 parts

A 63-year-old man is brought to the clinic by his wife who is concerned that he is in the early stages of senile dementia, Alzheimer type. The patient has been having memory disturbances lately and his wife reports that he is "not being as sharp intellectually as he used to be." Cognitive testing reveals that the patient has some deficiencies in memory and calculation. To test the patient's abstract thinking, which of the following is the most appropriate evaluation? A. Copy a figure B. Give the current date C. Interpret a proverb D. Name the past 5 presidents E. Subtract from 100 by increments of 7

The correct answer is C. Abstract thinking is defined as the ability to appreciate the symbolic or metaphoric meaning. It is especially relevant to this patient, given his high educational level. Interpretation of a proverb tests this ability. When assessing mental status with suspected dementia or memory problems, assess patients for the ability to communicate, insight into recent and past events, and recall. The Folstein Mini-Mental Status Examination screens for cognitive deficits and assesses orientation, registration, attention and calculation, recall, and language. Abstract thought can be assessed by having the patient name similarities and differences between objects and by interpretation of proverbs.

A 10-year-old boy comes to the health care provider with his parents at the request of school authorities. In the past couple of months the teachers have noticed that he has become more fidgety in class and at times coughs or produces strange noises. He also makes sudden movements and touches his peers in the middle of conversation. He has always had a blinking-type tic that has recently worsened. The main reason the parents were asked to bring him in for evaluation was the boy's sudden screams, which distract and scare the other students in class. He is academically successful, very neat, with no other behavioral or emotional problems. He is initially silent in the health care provider's office and then suddenly produces "the noise." He seems very upset when it occurs and admits that he is aware that other children make fun of him and avoid him because of it but he cannot control it. Which of the following is the most likely diagnosis? A. Attention deficit disorder B. Autistic disorder C. Gilles de la Tourette disorder D. Huntington disease E. Oppositional defiant disorder

The correct answer is C. This boy most likely has Tourette disorder, which belongs to the group of tic disorders with onset before age 18. It is more common in boys. Multiple motor and vocal tics are present during the illness many times a day, every day for more than a year. The patient must not have had a tic-free period for more than 3 months. The disturbance is not caused by a general medical condition or effects of substance. Coprolalia and echolalia typically occur later. Patients with Tourette syndrome have an early childhood onset with a worsening of symptoms in late childhood and early adolescence. Symptoms of Tourette syndrome include stereotypical repetitive movements that are accompanied by an urge to perform these movements. There are stereotypical repetitive phonic sounds accompanied by a need to perform those sounds. These patients typically will have an otherwise normal neurologic examination. Symptoms typically worsen under stressful conditions and typically improve when the patient is focused on other tasks.

A 30-year-old woman comes to her primary care provider with multiple symptoms of depression. She complains of about 2 months of feeling sad and anxious. She describes difficulty falling asleep at night, decreased appetite with a 15-lb weight loss, diminished interest in her hobbies, and poor concentration. The patient denies any other problems. Physical examination is within normal limits. The patient and her health care provider discuss initiating treatment with an SSRI. Which of the following would be the most important additional information to gather before starting the SSRI? A. Family history of depressive episodes B. Family history of alcoholism or drug dependence C. Past history of hypomanic or manic episodes D. Past history of kidney diseases E. Past history of obsessions or compulsions

The correct answer is C. Although all of the choices are valuable pieces of further information, a past history of hypomanic or manic episodes is the one choice that might significantly alter treatment planning. It is likely that all antidepressants, including SSRIs, have a potential to cause a switch into hypomania or mania, or to accelerate cycling in patients who have bipolar disorder. If the patient has a history of hypomanic or manic episodes, her depression would be considered a bipolar depression rather than a unipolar depression. In this case, the patient would likely need to be started on a mood stabilizer before beginning treatment with an antidepressant medication. Patients who have major depression who are treated with traditional antidepressant agents may develop a manic or hypomanic episode as a result of the use of these agents. If patients have hypomania or mania in their history, the patient's mood should be stabilized with a mood stabilizer prior to starting a traditional antidepressant.

A 26-year-old graduate student feels depressed, suicidal, and tense, and admits to "hurting herself" for many years without the intention to die. She explains that the pain she inflicts upon herself is therapeutic, helping her to release tension. She has several scars on her arms that were caused by superficial cuts made with a razor blade. She has 2 new superficial cuts from the night before. She believes that the current "crisis" is related to a recent breakup with a man, even though she was the initiator in the fight. She has had several stormy relationships with men, which typically end poorly. She goes directly into new relationships because she doesn't like being alone. Many times she feels empty and bored, yet she does not have the initiative to change things about herself. She frequently uses drugs to elevate her mood. She attends her classes regularly and usually is able to prepare for examinations. Which of the following is the most likely diagnosis? A. Adjustment disorder with depressed mood B. Atypical depression C. Borderline personality disorder D. Dysthymic disorder E. Narcissistic personality disorder

The correct answer is C. Borderline personality disorder is a pervasive pattern of instability in interpersonal relationships that begin in early adulthood. Efforts to avoid abandonment, a pattern of unstable relationships alternating between idealization and devaluation, unstable self-image, impulsivity, recurrent suicidal behavior, chronic feelings of emptiness, inappropriate intense anger, transient paranoid ideation, and affective instability are typical symptoms of this disorder. Patients who have borderline personality disorder typically have unstable and intense mood states with a history of multiple relationships wherein patients tend to see others as either all good or all bad. Patients who have borderline personality disorder tend to have impulsive, recurrent self-destructive behaviors such as cutting, substance abuse, drug overdose, and sexual promiscuity. These patients may also have episodic, intense anger states.

A patient is seen on the psychiatry consultation liaison service by a health care provider. To assess cognitive functions, the health care provider asks the patient what the proverb "Don't cry over spilled milk" means. The patient answers that if you spill what you cook, you have to do it all over again. Which of the following types of thinking is this patient most likely exhibiting? A. Abstract thinking B. Blocking C. Concrete thinking D. Confabulation E. Magical thinking

The correct answer is C. Concrete thinking is described as literal thinking that shows a lack of understanding of the nuances of meaning. These individuals lack the ability to use metaphors. Abstract thinking (choice A) refers to the ability to appreciate nuances of meaning and the ability to use metaphors and hypotheses appropriately. Blocking (choice B) is a disturbance in thought form, characterized by an abrupt interruption in the train of thought before the thought is finished. After a pause, the person is unable to recall what was being said. Confabulation (choice D) is defined as the unconscious filling of gaps in memory by imagined or untrue experiences that the patient believes are true, even though they are not based on facts. It is associated with organic pathology. It is classically described in a patient who has Korsakoff psychosis, in which the patient is not able to form new memories so the patient makes up a story to cover up for the lack of memory. Magical thinking (choice E) is a form of thinking in which thoughts, words, or actions assume power, such as causing or preventing events. It is typical for the preoperational phase of thinking in children. In the assessment of mental status of a patient, several aspects are tested. Patients should be assessed for ability to communicate, insight into recent and past events, and recall. The Folstein Mini-Mental Status Examination screens for cognitive deficits and assesses orientation, registration, attention and calculation, recall, and language. Abstract thought can be assessed by having the patient name similarities and differences between objects and by interpretation of proverbs. If the patient does not have the insight to comprehend the meaning of a proverb or metaphor, the patient is exhibiting concrete rather than abstract thinking.

A man who appears to be in his 20s is brought to the emergency department by the police after being found wandering around a local airport. The patient claims not to know how he got to this city and states repeatedly that everything he sees in the environment is unfamiliar. He has no identification or baggage and cannot remember his name. He is well-groomed and in no apparent distress. His process of thought is logical and he denies any mood complaints or perceptual disturbances. Physical examination, metabolic and toxicology screenings, neuroimaging, and electroencephalography are all within normal limits. Which of the following is the most likely diagnosis? A. Adjustment disorder B. Bipolar disorder, most recent episode manic C. Dissociative fugue D. Dissociative identity disorder E. Major depressive disorder

The correct answer is C. Dissociative fugue is a rare dissociative disorder involving sudden travel and amnesia about one's identity or the circumstances preceding arrival at one's destination. The etiology is unknown, though a correlation with recent life stressors may exist. The condition is usually self-limited. Patients are clear mentally except for who they are and how they arrived at the place where they are found to be lost. Dissociative fugue disorder is a condition in which patients will have prolonged psychogenic amnesia with a sudden loss of personal identity and with events regarding the personal past. At the same time, patients can recall other memories and learn new material; they may be able to function at a high level as long as their identity is not being threatened by their lack of remembering who they are and how they got to a certain point in their lives. These episodes last hours to days but could last weeks to months wherein the person may take on a new identity. Upon recovery, the patient has residual amnesia for the fugue state.

A 73-year-old man with benign prostatic hyperplasia and no past psychiatric history comes to the health care provider for a routine visit. His physical examination and routine laboratory studies are normal. The patient's wife died approximately 2 months ago after an extended course of colon cancer and since then, he has been "sad and lonely." He also indicates that he just recently began to recover weight that he lost in the days and weeks since his wife's passing. His sleep is also recovering and he has been spending more time with his children and grandchildren. He is troubled, however, by the feeling that he can actually hear his wife calling out his name when he is alone in the house that the couple shared for their entire marriage. He denies suicidal thoughts. Which of the following is the most appropriate next step in management? A. Electroencephalogram B. Short course of antipsychotic medication C. Reassurance that this is a normal grief reaction D. Recommendation for neuropsychologic testing E. Immediate psychiatric evaluation

The correct answer is C. This is a normal grief reaction, even the symptom of hearing one's own name being called by the departed. Additionally, the patient appears to be improving with respect to neurovegetative symptoms. Beyond grief counseling at the patient's own request, there is no further intervention that is absolutely necessary at this time. Major depression and potential suicidal ideation are the most important conditions to rule-out in this scenario. Since the patient has improvement in his mentation and functioning and denies suicidal thoughts, major depression and suicidal thoughts are less likely diagnoses. Grief reaction patients may have unusual symptoms (such as hearing the dead person's voice) that in other contexts would be considered to be psychotic in nature (auditory hallucinations) but are considered part of the normal reaction to a stressful episode. With time as a healer, symptoms will improve, with the need for antidepressant therapy based on how long and how disabling the symptoms continue.

A 28-year-old man comes to the emergency department complaining of a 3-month history of fatigue, depression, insomnia, and weight loss. He states he also has had difficulty with feelings of guilt over his occasional intravenous heroin use. On a Mini-Mental Status Examination, he has some mild cognitive defects, scoring 24 of 30. He is married and has always used condoms during sexual intercourse. Which of the following is the most appropriate next step in management of this patient? A. Order fluoxetine (Prozac) B. Order imipramine (Tofranil) C. Order an HIV test D. Order a urine drug screen E. Refer the patient for outpatient drug rehabilitation

The correct answer is C. Given this patient's risk factors for HIV, this is the most likely diagnosis in the context of mild cognitive deficits and mood disturbance. To begin treatment with antidepressant medication, mood disturbance caused by a general medical condition must be ruled out. Ordering fluoxetine (choice A) or imipramine (choice B) is thus inappropriate at this time. Risk factors for HIV infection include IV drug use, multiple sexual partners (especially if barrier methods are not used during sexual activity), percutaneous needle stick injury, and high maternal viral load (mother to child transmission). Screening should be done in high-risk patients or for anyone requesting the test. Both pretest and posttest counseling should be done when HIV screening is performed.

A 45-year-old woman comes to her primary care provider because of blurred vision. She states that this symptom started about 2 days ago. She denies any past history of significant medical or neurologic problems. She does state that several days ago she started treatment for depression with a psychiatrist because of a 2-month period during which she had several depressive symptoms. On review of systems, she admits to having increased her water consumption over the last several days because of a dry mouth. She also complains of dizziness when she stands up from lying or sitting. Her temperature is 37º C (98.6º F), blood pressure lying down is 135/75 mm Hg, blood pressure standing is 110/64 mm Hg, pulse lying down is 84/min, pulse standing is 95/min, and respirations are 16/min. Physical examination is unremarkable except for mild pupillary dilation. Which of the following medications most likely accounts for this patient's symptoms? A. Bupropion (Wellbutrin) B. Citalopram (Celexa) C. Imipramine (Tofranil) D. Nefazodone (Serzone) E. Sertraline (Zoloft)

The correct answer is C. Imipramine is a tricyclic antidepressant (TCA) that inhibits norepinephrine and serotonin reuptake. Like most TCAs, imipramine also has antagonistic effects at muscarinic, histaminic, and α-adrenergic receptors. This patient's complaint of blurred vision is most likely caused by the antimuscarinic effects of the medication. The blockade of muscarinic acetylcholine receptors causes mydriasis (pupillary dilation) resulting in blurred vision. Dry mouth is also caused by the anticholinergic effect of imipramine. Orthostatic hypotension is caused by the α1-adrenergic receptor blockade associated with TCAs. Multiple agents are effective in treating major depression. The choice of which agent to use may be based upon side-effect profiles. Although effective, tricyclic antidepressants are not first-line antidepressants because of their high anticholinergic properties (weight gain, dry mouth, urinary retention) and because of their high lethality based upon prolongation of the T wave on EKG and increased risk for torsades de points and ventricular tachycardia.

Which of the following disorders is most likely to be missed even by someone experienced in managing patients with psychiatric disorders? A. Delirium B. Dementia C. Fugue D. Somnambulism E. Stupor

The correct answer is C. In fugue, the person has an abrupt change of geographic location or identity without alterations in consciousness or memory changes. Dissociative fugue is one of the amnestic states that one can develop. Patients who have dissociative fugue states leave an area and assume a new identity in another location. Except for the concept of self, the patient is fully alert and able to answer questions appropriately. Because the patient is acting appropriately, it is difficult for a health care provider to identify this condition unless the patient's previous identity was known.

A 37-year-old woman with a history of bipolar disorder comes to her health care provider for a medication check. She has been taking the same dose of lithium carbonate daily for over 3 years. She is in generally good health, does not smoke, and has not had a manic episode for several years. She uses a diaphragm and condoms for birth control and has no plans to get pregnant in the near future. Given this patient's history, the values for which of the following routine lab tests are most likely to have remained the same over time? A. Serum blood urea nitrogen level (BUN) B. Serum creatinine level C. Serum lithium level D. Thyroid stimulating hormone level (TSH) E. White blood cell count

The correct answer is C. In general, lithium reaches a steady state level in the bloodstream once a dosage is maintained over a given period of time, and its level does not change unless conditions exist or medication changes that affect drug level interferes with this equilibrium. Lithium is a medication that is typically used on a long-term basis as a mood stabilizer. Known adverse effects of lithium include goiter, hypothyroidism, hyperparathyroidism, pseudotumor cerebri, and polyuria. Monitoring parameters for patients on lithium include creatinine and urinalysis at baseline and electrolytes for sodium monitoring, and CBCs can also be done to assess for leukocytosis.

The mother of a 23-year-old man brings her son to a mental health professional for a second opinion 1 week after his discharge from the hospital. The young man had been hospitalized for 1 week because of the new onset of auditory hallucinations and tangentiality of thoughts. During his hospitalization, he was diagnosed with schizophreniform disorder and was treated with risperidone (Risperdal) 2 mg PO BID. A medical workup for this first episode of psychosis was negative. In discussing course and prognosis with the patient and his mother, which of the following features would be considered a good prognostic indicator? A. Family history of schizophrenia B. Lack of any known precipitating factors or acute stressors C. Onset of psychotic symptoms within a month of the first change in behavior D. Poor premorbid educational and occupational performance E. Withdrawn, autistic behavior and flat affect

The correct answer is C. In schizophreniform disorder or schizophrenia, several features are associated with better long-term prognosis. The onset of psychotic symptoms within 4 weeks of the first noticeable change in usual behavior or functioning is one such feature. Other good prognostic features include: confusion or perplexity at the height of the psychotic episode, good premorbid social and occupational functioning, and the absence of blunted or flat affect. Schizophrenic disorders are typically chronic and even lifelong psychotic disorders. Patients who have better premorbid levels of functioning prior to the onset of schizophrenia typically will have better prognosis. Patients who have positive symptoms (such as active hallucinations) tend to respond to treatment better than patients who have negative symptoms (lack of interaction with others).

A 70-year-old woman is brought to the health care provider by her daughter because of personality change and forgetfulness for the past 3 months. Her last medical checkup was 1 year earlier, when she appeared in good spirits and excellent health. Six months ago her husband died. During the examination, the patient answers the health care provider's questions with "Yes," "No," or "I don't know," and appears apathetic and indifferent. When asked questions testing short-term memory, she often replies, "I don't know." Physical examination is unremarkable. Blood studies are normal, and TSH is within normal range. The health care provider arranges for an MRI examination of the brain. The T1-weighted image is shown below. Which of the following is the most likely diagnosis? A. Alzheimer dementia B. Brain tumor C. Normal pressure hydrocephalus D. Pick dementia E. Pseudodementia

The correct answer is E. The patient's MRI scan does not reveal any significant cortical atrophy. Cortical atrophy manifests with narrowing of gyri and widening of sulci. As a secondary effect, enlargement of the ventricles takes place (known as hydrocephalus ex vacuo). The ventricles are normal in this case. The patient's behavior, lack of interest, and verbal replies suggest a diagnosis of depression. Elderly people who have this condition often present with cognitive deficits, which may constitute the predominant clinical manifestation. Depressed patients tend to reply, "I don't know" to questions aimed at testing memory skills. A diagnosis of pseudodementia is applicable whenever severe cognitive deficits are not explained by objective findings by clinical, radiologic, or pathologic investigations. Depression is certainly an important cause to consider in the differential diagnosis of dementing disorders. Patients who have major depression may manifest symptoms consistent with depression. Depression should be screened for, because correction of this underlying condition will result in resolution of improvement of the patient's so-called "dementia" symptoms.

A 36-year-old woman comes to a health care provider for an initial appointment after relocating from her home several states away because of a transfer of her job. She is on lithium for bipolar disorder, and has been stable for several years after two manic episodes in her early 30s. She was previously seen by a psychiatrist every 2 months and had blood drawn for routine monitoring related to her lithium treatment every 6 months. Because it has been about 8 months since her last appointment, the health care provider decides to order lab work and to see the patient back in 1 week to finish gathering history, to review the laboratory results, and to provide the patient with another prescription. Which of the following laboratory studies would be most appropriate for the health care provider to order? A. Liver transaminases B. Platelet count C. Thyroid-stimulating hormone D. Urine culture E. Vitamin B12 level

The correct answer is C. Lithium affects thyroid function, and thyroid-stimulating hormone (TSH) levels should be monitored every 6-12 months. Lithium often causes a generally benign and often transient decrease in the concentration of thyroid hormones. About 30% of patients receiving long-term treatment with lithium will have elevated TSH levels. If symptoms of hypothyroidism emerge, treatment with levothyroxine may be indicated. During routine monitoring of lithium maintenance treatment, lithium plasma concentration and serum creatinine should also be measured periodically. Lithium is a medication used as a mood-stabilizing agent for long-term treatment in the management of bipolar disorder. This medication has multiple effects including EKG changes and also making the kidney insensitive to vasopressin, which results in a nephrogenic diabetes insipidus. Thyroid disease may also occur, so patients should be monitored for the development of hypothyroidism caused by lithium use.

A 29-year-old woman comes to her new primary care provider requesting medication to help with her recent spells of anxiety and depression. She wishes to be started on a medication that will not cause too much sedation. Examination of the patient's old records reveals 2 previous suicide attempts by overdose, once with acetaminophen at age 16 and another with aspirin during college. She is otherwise healthy and does not smoke cigarettes or drink alcohol. She is not currently taking any medication. Given this history, which of the following antidepressant medications would most likely be contraindicated for this patient? A. Buspirone (Buspar) B. Fluoxetine (Prozac) C. Nortriptyline (Pamelor) D. Paroxetine (Paxil) E. Sertraline (Zoloft)

The correct answer is C. Nortriptyline is a tricyclic antidepressant, which is lethal in overdose and is the leading cause of overdose-related deaths in the psychiatric population. Overdose is associated with prolongation of the QT interval, leading to cardiac arrhythmia and death. Given this patient's history of previous overdoses, a tricyclic antidepressant would not be indicated. Patients who have a history of major depression and previous suicide attempt should be preferentially treated with agents other than tricyclic antidepressants because of their cardiac-related toxicity as a result of prolongation of QT interval.

A 35-year-old engineer with a history of heroin abuse has been having marital problems because of his addiction. His wife insists that he needs treatment and that unless he deals with his addiction, she will file for divorce. The patient has been successful and has stayed clean for several years but relapsed recently when he ran into one of his old "drug buddies." In the meantime, he developed a fever and a cough and is admitted to the hospital for the treatment of pneumonia. While in the hospital he develops cramps, diarrhea, frequent yawning, and stays wrapped in a blanket because of chills and body aches. While waiting for the substance abuse consult, which of the following medications would be most useful for treating his autonomic symptoms? A. Alprazolam (Xanax) B. Clonazepam (Clonopin) C. diazepam (Valium) D. Flumazenil (Romazicon) E. Methylphenidate (Ritalin)

The correct answer is C. Of the medications listed, the only one that would actually help relieve the autonomic symptoms of withdrawal would be diazepam (Valium) for muscle cramps and relaxation. Other medications can be given for specific symptoms: loperamide for diarrhea, promethazine for vomiting, dicyclomine for abdominal cramping, and ibuprofen for muscle pain. Heroin withdrawal may be accompanied by autonomic symptoms such as chills, piloerection, abdominal pain and cramping, and agitation. Treatment of these autonomic symptoms is best accomplished by muscle relaxants such as diazepam (Valium).

An 82-year-old nursing home resident with moderate dementia secondary to Alzheimer disease has been agitated for the past month. She has been accusing the staff at the nursing home of stealing her belongings. Over the past 2 weeks she has also become physically aggressive toward the staff. Which of the following is the most appropriate management? A. Administer low doses of benzodiazepines when the patient becomes agitated B. Begin a trial of a low dose of an antidepressant medication, such as fluoxetine (Prozac) C. Begin a trial of a low dose of an atypical antipsychotic medication, such as risperidone (Risperdol) D. Recommend insight-oriented psychotherapy E. Recommend the frequent use of physical restraints as part of a behavior modification regimen

The correct answer is C. Patients who have senile dementia, Alzheimer type often develop paranoid delusions and agitation in the course of their illness. Memory impairment is a specific risk for the development of delusions that people are stealing from the patient, as patients misplace or lose their belongings. The treatment of choice for agitation and paranoia in the elderly demented patient includes low doses of atypical antipsychotic medication. Patients who have Alzheimer dementia can have behavioral manifestations such as aggressive behavior. Other patients who have this disease can start to withdraw ("Alzheimer apathy"). Patients who have Alzheimer dementia with aggression may respond to low-dose antipsychotics as a first-line treatment.

Which of the following best describes the attributes of individuals who commit elder abuse? A. They are concerned caretakers of the abused person B. They expect nothing in return from the abused person C. They have lived with the abused person for a long period of time D. They have no history of being abused E. They occasionally use alcohol/drugs in a recreational manner

The correct answer is C. Persons who abuse elderly people usually have lived with the elderly person for some time. There has been opportunity for idiosyncratic behavior on the part of the elderly person to create resentment in the caretaker. Elder abuse is usually not the expression of a momentary impulse; rather, it develops over some time. Families that have older adults and very young children living in the same household tend to experience high levels of stress, and this tension increases the risk for elder abuse to occur. Elder abuse can include neglect, physical, psychologic, sexual, or financial abuse. Risk factors for this condition include aging (along with frailty), dementia, dependence on the caregiver for personal care, financial dependence of the caregiver, and long-term dependence on the caregiver. Providers need to continue to have a high index of suspicion for identifying this condition.

A disgruntled surgical technician who is being treated for depression with phenelzine (Nardil) takes an analgesic he found on the drug cart in the surgical suite. Several hours later, he arrives at the emergency department in a delirious state and has a temperature of 40.4°C (105°F). Soon after his arrival, he begins having convulsions. Which of the following analgesics did he most likely take? A. Buprenorphine (Subutex) B. Codeine C. Meperidine (Demerol) D. Pentazocine (Talwin) E. Enalapril (Vasotec)

The correct answer is C. The combination of an MAO inhibitor (such as phenelzine) and meperidine (Demerol) produces a severe drug reaction. Many types of reactions have been reported, including serotonin syndrome, delirium, hyperpyrexia, convulsions, and hypertension. Other patients react as if they have had an overdose of meperidine (Demerol) and present with severe respiratory depression, cyanosis, hypotension, and coma. Meperidine (Demerol) is contraindicated in patients who have taken MAO inhibitors during the previous 14 days. Many drug interactions are possible with the use of MAO inhibitors. MAO inhibitors are never first-line agents because of these serious reactions that can also occur with certain tyramine-containing food. If a patient takes an MAO with meperidine, the patient is likely to have serotonin syndrome plus additive and synergistic effects with the use of this agent.

A 23-year-old woman is brought into the emergency department by ambulance at midnight, after her roommate found her using a paper clip to make lacerations on her wrists. The patient had been seen in the same emergency department twice during the past 2 weeks with a complaint of feeling very depressed and at times suicidal. She has a history of one drug overdose on children's vitamins. In both presentations to the emergency department, she was prescribed benzodiazepines for anxiety and sent home with an outpatient psychiatric clinic referral. On this presentation she states that if she is not admitted to the hospital she will go out and find a bridge to jump off. Which of the following is the most appropriate initial step in management? A. Admit the patient to the medical ward and await psychiatric consultation in the morning B. Explain to the patient that her attempts to mutilate herself will not result in suicide, and that she needs outpatient psychotherapy C. Order an emergency psychiatric assessment D. Remove the patient from hospital grounds by security escort E. Set up a new outpatient psychiatric clinic appointment

The correct answer is C. The fact that this patient has a history of a suicide gesture and expresses present suicidal ideation puts her at risk for harming herself. She should be kept in the hospital, at least until a psychiatric assessment can be made. A patient who has a history of a suicide gesture and current suicidal ideation is at risk for harming him/herself. Immediate psychiatric evaluation is called for, most likely followed by admission to the psychiatric unit.

A 1,400-g infant born at 35 weeks' gestation is 42 cm in length and has a head circumference of 28 cm. One day after birth, she becomes very irritable, tremulous, and inconsolable. Her cry is high-pitched. Her pulse is 174/min. There are no dysmorphic facial features. Her mother had inconsistent prenatal care and has a history of multiple inpatient hospitalizations for substance overdoses. To which of the following substances was this newborn most likely exposed in utero? A. Alcohol B. Barbiturates C. Cocaine D. Marijuana E. Opiates

The correct answer is C. The most commonly abused drug by pregnant mothers is cocaine. Infants are usually small for gestational age (SGA) and sometimes have microcephaly and neurodevelopmental abnormalities. Exposed infants are very irritable and inconsolable in the withdrawal period. Their cries are often high-pitched. They are also at increased risk for sudden infant death syndrome (SIDS). Periventricular leukomalacia (a CNS ischemic lesion) is also associated with cocaine exposure. Cocaine use in the United States occurs in approximately 2% of the adult population. Infants born to cocaine-addicted mothers are small in gestational size (probably because of vasoconstriction of the arteries supplying the fetus in utero), have neurodegenerative abnormalities, have microcephaly, and can have seizures. Detoxification procedures should be done to support the infant after birth during this withdrawal period.

A 30-year-old woman experiences the sudden onset of a headache, dizziness, and tingling in her left arm while standing on the subway. She is overwhelmed by fear and feels like she is going to die. She has difficulty catching her breath and is embarrassed to find that she is sweating profusely. She feels as if the episode is lasting forever, however, once she finds a seat she is able to recover. Over the next few weeks, she has several similar episodes in which she feels she will die. She develops tremendous anxiety when she needs to go somewhere alone that is farther than a couple of blocks from her house. She goes to the emergency department 2 more times because of similar attacks, but all diagnostic studies are normal. She has always been healthy and has never used alcohol or drugs. Her father died suddenly from a stroke 5 months ago while he was staying in her apartment. Which of the following is the most likely diagnosis? A. Dependent personality disorder B. Depersonalization C. Panic disorder D. Seizure disorder E. Somatization disorder

The correct answer is C. The patient has panic disorder, characterized by recurrent attacks of sudden discomfort in which she develops 4 or more somatic symptoms that last briefly and are accompanied by fear of dying or losing control. The symptoms are recurrent, last at least 1 month or more, and lead to change in behavior because of other criteria, such as anticipatory anxiety that causes avoidant behavior. The symptoms are caused by specific medical conditions or are due to the effects of substance abuse (e.g., drug, medication). Because of the unpredictable nature of when a panic attack could occur, these patients often become reclusive and can go on to develop agoraphobia. Panic disorder is characterized by recurring panic attacks along with a fear of future panic attacks that occurs over at least a 1-month period. To fit the criteria, panic attacks and thoughts surrounding future attacks must impact the patient's functioning. During the typical panic episode there is a sudden onset of intense physical and cognizant symptoms of anxiety which may have an identifiable trigger or may occur spontaneously. Because of the overwhelming fear of recurrent panic, patients may go on to develop agoraphobia.

A 27-year-old man with schizophrenia, chronic paranoid type comes to the clinic for a follow-up visit. He has been stably maintained on risperidone (Risperdal) 3 mg twice a day for a few months, but the dose was recently changed to 6 mg twice a day when he was hospitalized for pneumonia. In the clinic he is alert, oriented, well related, and his thought process is completely organized. He is not complaining of any auditory hallucinations and does not seem to be responding to internal stimuli. He has psychomotor agitation and has difficulty staying in his chair during the interview. When specifically questioned about the agitation, he says that he constantly feels like he has to move around. He denies any suicidal or homicidal intent, ideation, or plan. Which of the following is the most appropriate next step in management? A. Add an antidepressant B. Add haloperidol to control psychotic agitation C. Decrease the dosage of risperidone D. Increase the dose of risperidone because of incomplete treatment response E. Recommend electroconvulsive therapy for refractory psychosis

The correct answer is C. The patient's presentation is one of akathisia, a sense of restlessness and psychomotor agitation that patients can experience while taking antipsychotic medication. The atypical antipsychotic medications (olanzapine [Zyprexa], quetiapine [Seroquel], ziprasidone [Geodon], risperidone [Risperdal]) all confer less of a risk for the development of akathisia. With risperidone in particular, however, higher dosing is associated with an increased risk for the development of akathisia. The patient in question most likely had an escalation in risperidone dose while hospitalized secondary to agitation, which occurred during a delirious episode as a result of pneumonia and possible sepsis. Maintaining the patient on this dosage indefinitely, especially given these side effects, is unnecessary. Risperidone (Riperdal) is an atypical antipsychotic medication used in the management of schizophrenia. Side effects include extrapyramidal effects, tardive dyskinesia, and neuroleptic symptoms. Keeping the dose of risperidone to the lowest effective dose may limit the side effects. Extrapyramidal side effects may also be treated with diphenhydramine (Benadryl) or benztropine mesylate (Cogentin).

A 24-year-old man was recently hospitalized in a psychiatric unit, where he was started on olanzapine (Zyprexa). Upon visiting his new primary care provider for a routine physical examination prior to participating in a vocational rehabilitation program, his provider notices that his thoughts are quite illogical. For example, when she asks the young man what sort of employment he hopes to work toward, he answers, "If you subtract some yellow from the sky, it becomes greener." After a brief silence, he then states, "Telephone bills should never exceed twenty dollars...according to the Book of Numbers." Which mental status examination finding most accurately describes this patient's thought processes? A. Clang associations B. Concrete thinking C. Loose associations D. Tangential thoughts E. Thought blocking

The correct answer is C. This patient exhibits thought processes characterized by loosening of associations. His ideas are disconnected and seem to jump from one topic to an unconnected topic. Loose associations are one of the characteristic signs of a primary thought disorder, such as schizophrenia. Several abnormal thought processes are associated with schizophrenia, such as loosening of association, clang associations, tangential thoughts, thought blocking, and thought broadcasting.

A 12-year-old boy is brought to the health care provider because of a pattern of behavior that has been worsening over the past year. His mother states that he has been bullying other boys at school, staying out late without permission, setting small fires in abandoned lots, and physically abusing neighborhood cats. During the examination, a speculum from the examination room drawer falls out from underneath his shirt. Which of the following is the most likely diagnosis? A. Antisocial personality disorder B. Attention deficit hyperactivity disorder (ADHD) C. Conduct disorder D. Oppositional defiant disorder (ODD) E. Tourette syndrome

The correct answer is C. This patient has conduct disorder. The diagnosis requires a pattern of behavior that violates societal rules and the basic rights of others. Common features include lying, stealing, running away, staying out without permission, setting fires, truancy, vandalism, cruelty to animals, bullying, physical aggression, and sexual aggression. At least three of these features must be present to make the diagnosis. 25-50% of these patients go on to have antisocial personality disorder as an adult. Conduct disorder is defined as a repetitive pattern of behavior in which the basic rights of others or major age-appropriate societal rules or norms are violated, often with accompanying aggression to people and animals, destruction of property, deceitfulness, or theft. Patients who have this disorder may be diagnosed with antisocial personality disorder as adults.

Three hours after an uneventful appendectomy, a previously healthy (except for appendicitis) 78-year-old man becomes disoriented and confused. He repeatedly asks the nurses where he is, and his speech pattern is disorganized and rambling. His temperature is 37º C (98.6º F), blood pressure 120/80 mm Hg, pulse 70/min, and respirations 18/min. The patient is uncooperative but shows no physical abnormalities. Mental status examination is not possible because of an altered level of consciousness. Laboratory studies show no abnormalities. Which of the following is the most likely diagnosis? A. Adjustment disorder B. Brief psychotic disorder C. Delirium D. Delusional disorder E. Dementia

The correct answer is C. This patient has delirium, which is also called acute confusional state. It is very common in hospitalized and institutionalized elderly individuals. It is characterized by a rapid onset of impaired cognition, altered level of consciousness, disturbances in attention and psychomotor activity, and altered sleep-wake cycles. The symptoms tend to fluctuate and it is usually reversible when the underlying disorder is identified and treated. Common causes include psychologic and physical stress (for example, surgery), metabolic disturbances, neoplasms, infections, medications, cerebral and cardiovascular diseases, and withdrawal from alcohol and prescription medications. As opposed to dementia, the onset of delirium can definitely be identified because of its acute presentation. Delirium is an acute, fluctuating change in mental status that is associated with inattention and fluctuating levels of consciousness. It is a potentially life-threatening disorder. Delirium is the most common cause of hospital-related complications in the United States; it is seen most often in hospitalized elderly patients. By definition, patients need to have a disturbance in consciousness (reduced clarity of the environment), abrupt change in cognition, and acute presentation of condition that is caused by a general medical condition, substance abuse, or substance withdrawal.

A 32-year-old woman is brought to the emergency department by police for psychiatric evaluation. The officers found her in the street, naked and masturbating. She denies any recent drug use. She is, however, very irritable and has pressured speech. When questioned further, she states that she has had sex with 10 different men in the past 3 days, because she felt that she was "too beautiful to not share" her sexuality. She has been unable to go to work during these past days because of this 'need to share'. Which of the following is the most likely diagnosis? A. Dysthymic disorder B. Heroin abuse C. Mania D. Partial complex seizures E. Schizophrenia

The correct answer is C. This patient has evidence of hypersexuality, which is one of the many symptoms of mania encountered in patients with bipolar disorder. Other symptoms of mania include grandiosity, impulsivity, irritability, insomnia, and elevated mood. Patients with mania who are not under the influence of medications or intoxication may meet the classification of bipolar disorder on their presentation. Patients with acute mania may have impulsivity, reckless behavior, abnormally elated, euphoric or irritable mood, increased energy, and reduced need for sleep without feeling tired.

A 58-year old man is admitted to the trauma service after a motor vehicle accident that caused a fracture of his pelvis and right femur. His family reports that he recently lost his job because of poor work performance related to a worsening problem with alcohol. On day 3 of hospitalization (third postoperative day after internal fixation of the femoral fracture), the patient is noted to be disoriented. He tells the nursing staff about feeling and seeing snakes crawling in his bed. What is the most appropriate initial step in the management of this patient's altered mental status? A. Clonidine (Catapres) B. Haloperidol (Haldol) C. Lorazepam (Ativan) D. Naloxone (Narcan) E. Olanzapine (Zyprexa)

The correct answer is C. This patient is most likely experiencing alcohol withdrawal delirium (delirium tremens). This most severe form of alcohol withdrawal is a medical emergency that can result in significant morbidity and mortality. Untreated, delirium tremens has a mortality rate of approximately 20%. In addition to delirium, patients in this stage of withdrawal exhibit autonomic hyperactivity and fluctuating levels of psychomotor activity. About 5% of all alcohol-dependent hospitalized patients will develop delirium tremens. The condition frequently develops unexpectedly around the third hospital day. Benzodiazepines, such as lorazepam, are the treatment of choice. Lorazepam can be administered PO, IM, or IV. Supportive management, including hydration and very close monitoring, should also be initiated.

A 19-year-old college freshman feels that she cannot get out of bed in the morning because of a "feeling of heaviness" in her arms and legs. She is struggling academically because of her lack of energy. Additionally, she feels especially "moody" and cries when her professors criticize her, and she is taking "way too long to get over this guy who said he wasn't interested in dating me." She eats and sleeps "way, way too much." Physical examination, thyroid testing, routine chemistries, CBC, and testing for Epstein-Barr virus are all negative. Which of the following is the most likely diagnosis? A. Borderline personality disorder B. Cyclothymic disorder C. Major depressive disorder, with atypical features D. Schizophrenia, chronic paranoid type E. Social phobia

The correct answer is C. This patient most likely has major depressive disorder, with atypical features. This diagnosis is considered to be a part of the major depression spectrum, but differs insofar as neurovegetative symptoms are concerned. Whereas patients who have classic/melancholic depression lose weight and have severe insomnia and early awakening, patients who have atypical depression eat and sleep in excess. Additionally, patients who have atypical depression are considered to be especially sensitive to rejection and display the "leaden paralysis" that the patient describes as "fatigue." Major depression is classically associated with insomnia, early morning awakening, inability to concentrate, poor eating, and anxiety. Atypical features include sleeping and eating too much, and this may be the presentation that some patients will have. The choice of SSRI may be different (e.g., fluoxetine), which may help to energize the patient, versus paroxetine, which would have more effectiveness in treating patients who have major depression with anxiety-type symptoms.

A 53-year-old female lawyer who has been married for 20 years comes to her health care provider's office because she has not felt "up to par" over the past 2 months. She is married, has 2 grown sons, and has a good practice. In the past couple of weeks, she has stopped taking care of her appearance and has frequently called in sick to work, when she actually has been having difficulty getting out of bed. She states that she has lost her appetite recently and her interest in sex with her husband has decreased considerably. She recently told her husband that at times she wonders whether she should go on living. She denies any history of drug or alcohol abuse, and a complete physical examination 3 months earlier showed her to be in good health. Which of the following is the most likely diagnosis? A. Bipolar I disorder B. Generalized anxiety disorder C. Major depressive disorder D. Panic disorder E. Schizophrenia

The correct answer is C. This patient most likely has major depressive disorder. She has had symptoms for 2 months, surpassing the criteria for a minimal length of depression of 2 weeks. Her other symptoms include loss of appetite, hypersomnia, decreased libido, loss of energy and interest in pleasurable activities, and vague suicidal ideation, all of which are criteria for major depression. Major depression is classically associated with insomnia, early morning awakening, inability to concentrate, poor eating, and anxiety. Assessment should always include risk assessment for suicide. Patients need to have symptoms for at least 2 weeks to meet the criteria, and this depression must impair the functioning of their lives in order to meet the criteria for major depression.

A 43-year-old woman comes to the emergency department complaining of dizziness, tremor, diaphoresis, and shortness of breath. She indicates to the health care provider that she has come into the hospital with similar complaints twice in the past several weeks. On those occasions, as today, her physical examination, routine laboratory studies, electrocardiogram, and cardiac enzymes have been unremarkable. The patient states that these episodes are starting to concern her greatly and she is worried about going out of her house alone. Which of the following is the most likely diagnosis? A. Factitious disorder B. Generalized anxiety disorder C. Panic disorder D. Schizophrenia E. Social phobia

The correct answer is C. This patient's symptoms are all suggestive of panic attacks. Her symptoms do not appear to be related to substance use or a general medical condition. The frequency of her symptoms and her agoraphobia (fear of social situations) are also indicative of a diagnosis of panic disorder. Panic disorder is characterized by recurring panic attacks along with a fear of future panic attacks that occurs over at least a 1-month period. To fit these criteria, the panic attacks and thoughts surrounding future attacks must impact the patient's functioning. During the panic episode, there is typically a sudden onset of intense physical and cognizant symptoms of anxiety that may have an identifiable trigger or may occur spontaneously. Because of the overwhelming fear of recurrent panic, patients may go on to develop agoraphobia. Agoraphobia is not just the fear of leaving a house or going into an open state but also includes situations in which the patient perceives that escape would be difficult.

A 26-year-old man comes to his primary care provider complaining that he has been more and more afraid to leave his house because he has been having frequent anxiety attacks that occur without warning. He is worried that he will have an attack at some time while he is in public. He has no other medical conditions. Physical examination is unremarkable. Which of the following is the most appropriate initial medication for this patient in an acute situation? A. Buspirone (Buspar) B. Chlorpromazine (Thorazine) C. Clonazepam (Klonopin) D. Mirtazapine (Remeron) E. Trifluoperazine (Stelazine)

The correct answer is C. This patient's symptoms suggest that he has panic disorder, which is most appropriately treated acutely with a benzodiazepine with a medium length half-life and duration of action, such as clonazepam. Patients who have panic attacks with agoraphobia are best treated with cognitive behavioral therapy. Medical treatment involves SSRIs or SNRIs as first-line treatment. Patients are also given benzodiazepines acutely, because it takes several weeks for the SSRIs and SNRIs to become effective. Buspirone, an anxiolytic, is not helpful for panic disorder because of the length of time it takes to get a response to the medication and the fact that it is not as effective as the other agents.

A young woman comes to a health care provider for help. During the interview, she reveals that she was raped when she was a teenager. She is currently dating a man with whom she would like to have sexual intercourse. When they tried, however, she felt "uptight," and they could not have intercourse. Which of the following is the most likely diagnosis? A. Dyspareunia B. Fetishism C. Frotteurism D. Pseudocyesis E. Vaginismus

The correct answer is E. Vaginismus is one of the sexual pain disorders. It is defined as involuntary muscle contraction of the outer third of the vagina that interferes with intercourse. It is prevalent in women who have a history of sexual trauma, emotional abuse, rigid religious upbringing, or psychosexual conflict. Vaginismus is either voluntary or involuntary contraction of the vaginal muscles around the introitus. It may result from pain, sexual trauma, or a negative feeling about the partner. It may result from previous sexual trauma. Vaginismus may result in dyspareunia, which is a condition in which there is painful intercourse.

An 18-year-old girl is brought to the clinic because she periodically pulls her hair and puts it in her mouth. She has several bald patches on both sides of her head. Initially, her mother thought it was some sort of infection but the dermatologist "did not find anything." The patient describes a sense of relief when she does it and claims that she really does it when upset, anxious, or overwhelmed. She admits that she has been doing it on and off for at least 2 years and thinks that it started after her parents got divorced. She is doing well in school. Which of the following is the most appropriate initial step in management? A. Benzodiazepine to relieve tension B. Tricyclic antidepressant C. Ruling out the presence of comorbid conditions D. Interpersonal psychotherapy E. Carbamazepine (Tegretol)

The correct answer is C. When trichotillomania is suspected, the most appropriate initial step is to rule out the presence of comorbid conditions. The most frequent ones would be obsessive-compulsive, borderline, and depressive disorders. When assessing a patient who has documented trichotillomania, it is important to understand that this behavior usually accompanies another behavioral disorder. Rule out other conditions before prescribing medication or psychotherapy. Otherwise, such treatment will not be of long-term help to the patient. First-line therapy for obsessive-compulsive disorder is SSRIs, which normally must be prescribed in higher doses than would be prescribed to treat major depression.

A patient is talking to his mental health care provider about a conflict he has with his partner. They argue because the partner complains that the patient is inefficient and procrastinates doing things that are his responsibility. He never completes a task the way in which he was asked to do it, but in his own way and usually much later. Which of the following types of defense mechanism is this patient exhibiting? A. Acting out B. Blocking C. Passive-aggressiveness D. Regression E. Splitting

The correct answer is C. With passive-aggressiveness, aggression toward others is expressed indirectly, usually through procrastination, stubbornness, passivity, and forgetfulness. People using this defense mechanism might take an unnecessarily long time to get ready for a party they do not wish to attend. Or a person may leave notes to avoid face-to-face discussions or confrontations. Passive-aggressive behavior is a defense mechanism that manifests as resentment, stubbornness, procrastination, sullenness, and repeated failure to carry out requested tasks for which that person is assumed to be responsible.

A 9-year-old boy comes to a new health care provider with his mother because of a significant decline in functioning and increased social withdrawal. He has always had difficulties at school and was timid and anxious when speaking in front of others. Looking back, the mother remembers that she had first noticed problems when he started at a new school after the family relocated because of her husband's new job. After a month in the new school, she had taken him to a pediatrician because of recurrent episodes of abdominal pain, nausea, and tiredness that caused repeated absences from school. All medical evaluations have been negative. Which of the following is the most likely diagnosis? A. Adjustment disorder with anxiety somatization disorder B. Factitious disorder C. Hypochondriasis D. Separation anxiety disorder E. Somatization disorder

The correct answer is D. Children who have separation anxiety disorder (school phobia) usually develop physical symptoms when they are separated from home or from people they are attached. The development of these symptoms is developmentally inappropriate. These symptoms consist of excessive anxiety evidenced by symptoms present by at least 4 weeks. The most common symptoms include repeated physical complaints, repeated nightmares, and excessive worry about losing significant figures, reluctance to attend school because of fear of separation, and fear that an untoward event will lead to separation from a major attachment figure. Symptoms are not caused by a general medical condition or other developmental disorder. Onset is before the age of 18. Many school-aged children fear going to school. The extreme of this problem is separation anxiety disorder in which symptoms include but are not limited to physical complaints, nightmares, worrying about parents and siblings when not in sight, and fear of being permanently separated from important people. Symptoms must be present for at least 4 weeks to meet the criteria for diagnosis.

A 39-year-old man with schizophrenia comes to the health care provider for a follow-up visit. He has been in a residential treatment program for several weeks, after being discharged from the hospital. While in the hospital he had been started on clozapine (Clozaril). He had initial complaints about sedation and increased salivation but was able to continue with the medication. His blood count is checked weekly and remains within normal limits. Today he complains of generalized weakness, fatigue, palpitations, chest pain, fever, and shortness of breath. His case manager confirms that he has not been feeling well throughout the whole week. His blood pressure, although on the lower side, is basically unchanged since the last visit. Which of the following side effects of clozapine should be considered as the most concerning, given this clinical picture? A. Hepatitis B. Hyperglycemia C. Hyponatremia D. Myocarditis E. Vertigo

The correct answer is D. Clozapine-induced myocarditis has been reported in several cases that ended fatally. Additional black-box warning regarding possibility of myocarditis should be provided for patients and their caregivers. Education about possible symptoms and signs should be provided. In this case the description of symptoms in a patient who has been on a stable dose of clozapine should raise a suspicion of possible myocarditis. Clozapine (Clozaril) is an atypical antipsychotic that is used in the management of resistant schizophrenia. It has several black-box warnings such as agranulocytosis, seizures, myocarditis, orthostatic hypotension with or without syncope, and dementia-related psychosis. The risk for the development of myocarditis is in the first month of use of this agent. Patients on clozapine should have close monitoring that includes serial CBCs, weight monitoring, serial fasting glucose levels, chemistry profiles, and possible baseline and serial electrocardiograms.

A 30-year-old woman with a history of tonic-clonic seizures complains of double vision, thickened gums, and growth of facial hair since starting a new medication. Which of the following anticonvulsant medications is most likely responsible for her symptoms? A. Carbamazepine (Tegretol) B. Ethosuximide (Zarontin) C. Phenobarbital (Phenobarb) D. Phenytoin (Dilantin) E. Valproic acid (Depakene)

The correct answer is D. Diplopia, gingival hyperplasia, and hirsutism are classic side effects of phenytoin. Other side effects include nystagmus, sedation, ataxia, and enzyme induction. Phenytoin is used in the treatment of grand mal and tonic-clonic seizures. It is not used for absence seizures. Phenytoin (Dilantin) use is associated with several well-known side effects such as gingival hyperplasia, hepatotoxicity, thrombocytopenia, leukopenia, agranulocytosis, and megaloblastic anemia when taken orally. Phenytoin results in severe cardiac adverse events when given intravenously, which can include ventricular fibrillation and cardiac conduction abnormalities.

A 27-year-old male prisoner with a self-described history of physical abuse is brought to the emergency department by prison staff. The patient states that he has severe leg pain after falling out of his bunk bed and that he is unable to walk. Neurologic examination shows normal deep tendon reflexes, but the patient has decreased sensation to pain and pinprick. The following morning the patient is seen walking, but complains that he is unable to pass urine, and he is told by the nurse that he will have to have a catheter inserted. As his health care provider walks in unexpectedly on rounds, the patient is seen sneaking back into bed from the direction of the restroom. Which of the following is the most likely diagnosis? A. Antisocial personality disorder B. Conversion disorder C. Drug dependence D. Factitious disorder E. Schizophrenia

The correct answer is D. Factitious disorder presents with physical symptoms that are consistent with true illness but are under the voluntary control of a patient. It is often dependent on a patient's need to fulfill the "sick role" and be under the care of a health care provider. Often, multiple invasive procedures or examinations are done at the request of the patient before a diagnosis is reached. There may be secondary gains as a result of these consciously produced symptoms. Factitious disorder is a condition in which patients unconsciously produce physical symptoms for the secondary benefits that can be attained. These benefits may include insurance or compensation reward. The patient voluntarily is aware that the symptoms are there for this secondary gain. Subsets of this condition are malingering (secondary compensation) and Munchausen syndrome (patient wants the attention of a sick role). There is no evidence that these symptoms are caused by real physical reasons after an appropriate evaluation is performed.

A 35-year-old woman is referred by her dermatologist to a mental health practitioner. For the past several years, she has had a compulsive need to wash her hands after shaking hands with someone. Because she is in the public relations business, hiding her problem while still performing her ritual has become increasingly difficult. She knows that if she doesn't do it, she becomes nervous and tense. Which of the following is the most appropriate treatment? A. Buspirone (Buspar) B. Clonazepam (Klonopin) C. Diazepam (Valium) D. Fluvoxamine (Luvox) E. Risperidone (Risperdal)

The correct answer is D. Fluvoxamine is an SSRI that has been approved for the treatment of obsessive-compulsive disorder (OCD). Its efficacy has been established in several trials. It is FDA-approved for the management of obsessive-compulsive disorder. Treatment of obsessive-compulsive disorder may be with cognitive behavioral therapy with or without pharmacotherapy. First-line therapy for reducing anxiety related to avoiding compulsive behavior is SSRIs. Other agents that have been approved for this condition include some tricyclics such as clomipramine (Anafranil). Pharmacotherapy for management of OCD is classically at higher doses than the doses that are used for management of major depression.

A 45-year-old woman returns to her psychiatrist for her routine biweekly appointment 2 months after being hospitalized for an episode of severe, recurrent major depressive disorder with psychotic features. During her hospitalization, she was started on 2 medications, an antidepressant and an antipsychotic, and she has continued these medications daily as an outpatient. At her appointment, she complains to her health care provider that she has missed her menstrual period for 2 months. She also complains of tenderness in her breasts, and an occasional small amount of milky discharge from her breasts onto her blouse. When questioned further, she also admits to low libido over the past month. Which of the following medications is most likely responsible for this constellation of symptoms? A. Olanzapine (Zyprexa) B. Paroxetine (Paxil) C. Quetiapine (Seroquel) D. Haloperidol (Haldol) E. Sertraline (Zoloft)

The correct answer is D. Haloperidol is a classic antipsychotic that can cause significant elevations in plasma prolactin levels. Conventional antipsychotics and risperidone can cause hyperprolactinemia because of their dopamine antagonism, which results in excessive amounts of dopamine being released. Clinical manifestations of hyperprolactinemia may include galactorrhea, sexual dysfunction, menstrual irregularities including amenorrhea, infertility, and weight gain. Typical antipsychotics can result in a great deal of dopamine inhibition. Dopamine in the CNS acts to inhibit prolactin secretion, so dopamine deficiency states would result in excessive prolactin release from the anterior pituitary. Signs of hyperprolactinoma include galactorrhea (only in females whose breasts are under the influence of estrogen), infertility, and decreased libido. Men who have hyperprolactinoma may have decreased libido and decrease in the size of the testicles.

A 57-year-old man comes to the clinic because of "depression." The chart states that he has been tearful and has experienced a 15-pound weight loss in the months following the death of his wife. Which of the following is the most appropriate initial question to open the interview? A. "Are you having problems with your sleep or appetite?" B. "Do you feel suicidal?" C. "Do you hear voices?" D. "What brings you here today?" E. "Why are you depressed?"

The correct answer is D. Interviews should be commenced with open-ended questions to allow the patient to express his present symptoms and reasons for coming in for treatment. Not only is it more interpersonally respectful to the patient, but also using open-ended questions at the beginning of patient interviews allows for the gathering of a more detailed and complete history of present illness. Remember that all interviews, not just psychiatric evaluations, should begin with an open-ended question such as, "What brings you here today?" This will allow the patient to explain in his own words why he is seeking treatment. Major depression is classically associated with insomnia, early morning awakening, inability to concentrate, poor eating, and anxiety. Suicide ideation should be assessed, as patients are at higher risk for self-harm. Choice of SSRI may vary; fluoxetine may help to energize the patient, while paroxetine would be more effective at treating major depression with anxiety-type symptoms.

A 17-year-old adolescent is brought to the hospital by police after being stopped for driving too slowly. He denies any use of alcohol, but the patient's movements seem slow. He is laughing inappropriately, complaining of being hungry, and has a dry mouth. His gait is somewhat slow and his coordination is impaired. There is significant conjunctival injection, which he tries to explain as being caused by pollen allergy. Which of the following drugs will most likely appear on a urine drug screen? A. Amphetamines B. Benzodiazepines C. Cocaine D. Opiates E. Tetrahydrocannabinol

The correct answer is E.Cannabis intoxication is usually characterized by sensitivity to external stimuli, subjective slowing down, impairment of motor skills (including problems with operation of motor vehicles), conjunctival injection, dry mouth, tachycardia, and increased appetite. Marijuana intoxication involves making the patient relaxed and calm with a slowness to respond and increased appetite (also known as the munchies). The patient's judgment may be impaired and the reaction time is slowed. There is no antidote to a patient who has an excessive intake of marijuana.

A patient who has a long history of bipolar disorder, mixed type, was recently started on an anticonvulsant that he was told would help with his mood swings. He had been on it for 5 weeks when he developed atypical targetoid lesions with focal confluence and detachment of the epithelium of about 10% of his body surface. Which of the following medications did this patient most likely take? A. Lithium (Lithobid) B. Clonazepam (Klonopin) C. Gabapentin (Neurontin) D. Lamotrigine (Lamictal) E. Valproate (Depakene)

The correct answer is D. Lamotrigine is approved as an adjunct in the treatment of refractory seizures. Its effectiveness in bipolar disorder has been demonstrated in open trials. The most serious side effect of lamotrigine is rash. When the dose is titrated up too quickly, the potentially fatal Stevens-Johnson syndrome can occur. Lamotrigine (Lamictal), a medication used with both bipolar disorder and seizure disorders, has known side effects involving rashes that can include Stevens-Johnson syndrome, toxic epidermal necrolysis, and even rash-related deaths. Adverse blood reactions may also occur, which can include aplastic anemia, hemolytic anemia, neutropenia, thrombocytopenia, and pure red blood cell aplasia.

A 35-year-old woman with bipolar disorder is brought to the emergency department because she is stiff and unable to swallow or talk. She is confused and tremulous. Her family is concerned because she developed a high fever last night, and they suspected that she developed an infection. She was recently treated in the hospital for a manic episode and received many haloperidol injections. She was discharged after she had received the injections and was started on lithium. In the past couple of days, her family reports that she has appeared "strange." Her temperature is 39.5 ºC (103.1 ºF), blood pressure 160/108 mm Hg, pulse 124/min, and respirations 24/min. She appears tremulous, diaphoretic, and confused. She has increased tone in her neck and extremities. Her leukocyte count is 21,200/mm3 and serum creatinine phosphokinase is 5,238 U/L. A thorough evaluation for infection is negative. Which of the following is the most likely diagnosis? A. Acute dystonia B. Lethal catatonia C. Malignant hyperthermia D. Neuroleptic malignant syndrome (NMS) E. Serotonin syndrome

The correct answer is D. NMS is a rare complication of neuroleptic therapy which confers high mortality if not recognized and treated promptly. It is defined by the development of severe muscle rigidity and elevated temperature in association with ≥2 of the following: dysphagia, tremor, diaphoresis, tachycardia, change in level of consciousness, leukocytosis, elevated or labile blood pressure, and elevated creatinine phosphokinase as an indicator of muscle injury. Predisposing factors include high neuroleptic doses, intramuscular injections, and lithium treatment. Neuroleptic malignant syndrome is a potentially life-threatening complication of treatment with an antipsychotic medication. It is associated with altered mental status, muscle rigidity, autonomic instability, and fever. Despite suspecting a patient of having neuroleptic malignant syndrome, patients should also be evaluated for sepsis and drug reactions. Treatment should consist of stopping the offending medication and supportive measures such as cooling and hydration. There is a higher incidence of this medication complication with the older, first-generation antipsychotic agents versus the newer dopamine receptor blocker antipsychotic medications.

A 48-year-old obese woman with a history of psychotic depression and polycystic ovary syndrome is brought to the psychiatric emergency department following a suicide threat. She is difficult to interview because she feels tired and sleepy and states she is on a continuous positive airway pressure (CPAP) machine at home secondary to having obstructive sleep apnea. She reports that she has been on fluoxetine (Prozac) and other medications, which she got for "the voices." Her medication list has been obtained off of her electronic medical record. During the interview, the health care provider learns she has gained 20 pounds in the past several months. Which of the following medications most likely caused this significant weight gain? A. Amoxapine B. Lorazepam (Ativan) C. Nefazodone (Serzone) D. Olanzapine (Zyprexa) E. Metformin (Glucophage)

The correct answer is D. Olanzapine (Zyprexa) is an atypical antipsychotic indicated for the management of psychotic disorders. Weight gain and type 2 diabetes have been associated with the use of this medication. Olanzapine (Zyprexa) is a second-generation, atypical antipsychotic medication that leads to weight gain, hyperglycemia, hyperlipidemia, and type 2 diabetes. It is used in the management of bipolar disorder and schizophrenia, and treatment-resistant major depression. It may also be used as adjunctive therapy with valproate or lithium to boost their effectiveness.

A 16-year-old boy is brought into the emergency department by his friends. He is semicomatose, with a pulse of 60/min and respirations of 6-8/min. His pupils are constricted. Which of the following will most likely be revealed on a urine toxicology screen? A. Amphetamine intoxication B. Cocaine intoxication C. Ethanol intoxication D. Opiate intoxication E. Tricyclic antidepressant intoxication

The correct answer is D. Opioids (e.g., heroin, codeine, methadone) can cause respiratory depression, coma, sinus bradycardia, and pupillary constriction. Death usually results from aspiration of gastric contents, respiratory arrest, or cerebral edema. Opiate abuse can cause respiratory depression, CNS depression, hypotension, bradycardia, syncope, shock, seizures, paralytic ileus, small, constricted pupils, sedation, and dry mouth. It can also cause visual disturbances, headache, and rash.

A heroin addict is found unconscious in an alley with an empty syringe beside him. When his blood gases are checked, which of the following would be expected? A. Metabolic acidosis B. Metabolic alkalosis C. Normal pH balance D. Respiratory acidosis E. Respiratory alkalosis

The correct answer is D. Opioids, such as heroin, depress respiration centrally by reducing the responsiveness of brainstem respiratory centers to CO2. The resulting hypoventilation leads to CO2 retention because of the inability of the patient to blow off the CO2. This increases the production of carbonic acid (H2CO3) by carbonic anhydrase present in red blood cells (which converts CO2 to carbonic acid). Dissociation of carbonic acid to bicarbonate (HCO3-) and protons produces a respiratory acidosis. Heroin intoxication causes depression of the respiratory system with resultant loss of drive to breathe. This results in respiratory acidosis caused by the lack of breathing. Naloxone (Narcan) can be used to treat opioid intoxication since it antagonizes various opioid receptors and acts as an opioid antagonist.

A 38-year-old woman comes to her primary care provider with complaints of fatigue and insomnia. Review of systems reveals no medical symptoms, but the patient does endorse several symptoms on the psychiatric review of systems. She describes feeling tired most of the day, having difficulty falling asleep as well as waking up too early, poor appetite, frequent crying spells, poor concentration, and a recent loss of interest in her hobbies. Physical examination and routine laboratory studies including thyroid stimulating hormone are within normal limits. Which of the following is the most appropriate initial pharmacologic intervention? A. Alprazolam (Xanax) B. Buspirone (Buspar) C. Imipramine (Tofranil) D. Paroxetine (Paxil) E. Phenelzine (Nardil)

The correct answer is D. Paroxetine is a selective serotonin reuptake inhibitor (SSRI). Along with other newer antidepressants, the SSRIs are currently the first-line agents for the treatment of depression. This patient has a symptom profile consistent with major depressive disorder, and treatment should be initiated with a goal of complete remission of symptoms. SSRIs are safe in overdose and have a relatively mild side-effect profile. SSRIs are considered to be first-line agents used in the management of major depression. They are highly effective and have a strong safety profile. Other first-line agents include SNRIs such as venlafaxine (Effexor) and duloxtine (Cymbalta) and agents that effect norepinephrine and dopamine such as buproprion (Wellbutrin).

A 54-year-old woman comes to her primary care provider for her routine yearly health maintenance examination. She denies any new problems. Her only significant medical history includes a 25-year history of schizophrenia, which has been well controlled with antipsychotic agents, and a 5-year history of hypertension for which she takes a diuretic. Vital signs and physical examination are within normal limits. The patient is noted to have occasional irregular puckering and lip-smacking movements, however. She denies having noticed the abnormal movements, and her speech is normal. Which of the following is the most likely diagnosis? A. Acute dystonia B. Akathisia C. Pseudoparkinsonism D. Tardive dyskinesia E. Tardive dystonia

The correct answer is D. Tardive dyskinesia (TD) is a syndrome characterized by abnormal choreiform and athetoid movements occurring late in onset in relation to initiation of antipsychotic treatment. TD usually develops after months to years of treatment with antipsychotic agents, and is presumably much less likely to occur with the use of atypical antipsychotics. Risk factors for TD include older age, longer duration of antipsychotic treatment, and presence of an affective disorder. The abnormal involuntary movements usually involve orofacial muscles but may include the trunk and extremities. Regular examinations should be done to ensure that patients treated with these medications do not develop signs of TD. Tardive dyskinesia is a known complication seen after long-term treatment with an antipsychotic agent. Symptoms consistent with tardive dyskinesia include choreiform movements of the lips, tongue, and mouth. The older antipsychotics are much more likely to cause this condition than the newer, atypical agents. When these symptoms occur, patients should have discontinuation of the offending agent or at least a change from a typical to an atypical antipsychotic agent.

A 12-year-old boy is brought to his health care provider because of failing grades in school. The boy failed last year and is repeating the sixth grade. His parents have suspected for several years that the boy may have low intelligence because of repeated difficulties in school. The family is referred to a psychologist for psychologic testing, including an assessment of intelligence. Which of the following tests would be most appropriate for evaluating the boy's intelligence and to assist with possible placement in special classes in his school? A. Millon Clinical Multiaxial Inventory B. Minnesota Multiphasic Personality Inventory C. Rorschach Test D. Stanford-Binet Test E. Thematic Apperception Test

The correct answer is D. The Stanford-Binet Test is a comprehensive intelligence test that is used in psychiatry and education. Another widely used intelligence test is the Wechsler Adult Intelligence Scale. Such tests are routine intelligence tests in the assessment of borderline intellectual functioning, mental retardation, and specific learning disorders. When a patient is assessed for intelligence quotient (or intelligence potential), the Stanford-Binet Test and the Wechsler Adult Intelligence Scale are two methods that are used. Results of this test can help to determine whether poor school performance is based on innate intellectual abilities or whether there are associated other entities involved with the drop in school performance.

A former drug abuser visits his health care provider to ask for pain medication for a back pain that has a true physiologic cause. The patient's previous history of drug abuse is taken into account when the health care provider plans the patient's treatment regimen. Which of the following medications used in this setting has the greatest potential for abuse? A. Codeine B. Dextromethorphan C. Loperamide (Imodium) D. Meperidine (Demerol) E. Nalbuphine (Nubain)

The correct answer is D. The drug that is most likely to be abused is that which causes the greatest euphoric effect. Euphoria is mediated by mu opioid receptors, and meperidine is the only drug listed that is a full mu agonist. Meperidine is rarely a good choice for pain control because of its rapid onset of activity (rapid peak) and of its inability to provide a sustained pain relieving response for an adequate time before the next dose is due. Meperidine (Demerol) has the potential to be abused because of its clinical effects. Pain mediation is rapid after a parenteral dose but length of activity does not reach the 3-hour window that is commonly required for repeat dosing. Additionally, meperidine has a toxic metabolite, normeperidine, that accumulates with ongoing use or with hepatic or renal impairment, and this metabolite lowers the seizure threshold.

A 63-year-old man with schizoaffective disorder is brought to the hospital by his health aide because of a 3-day history of severe weakness in his legs, an unsteady gait, and diarrhea. His hands are shaking and he seems mildly confused. He was recently hospitalized because of a manic episode and was placed on a combination of medications. While in the hospital, he was also diagnosed with chronic kidney disease, hypertension, and obesity. His medications were changed and he was placed on a combination of lithium and valproate (Depakene). Nifedipine (Procardia, Adalat) has been added to his antihypertensive medication. At this time, which of the following is the most appropriate initial step in management? A. Check to see if the patient has been drinking alcohol B. Discontinue nifedipine C. Obtain stool specimen for analysis D. Order blood lithium level E. Order blood valproate level

The correct answer is D. The first thing to do here is obtain the lithium level. This patient has been diagnosed with chronic kidney disease, and lithium is almost completely cleared in the kidney. The symptoms that the patient displays are consistent with adverse effects of lithium. Lithium is used in the management of schizoaffective disorders along with bipolar I disorder. It has a black box warning with regard to lithium toxicity and toxic accumulation can be a problem even at the high end of its therapeutic spectrum. Signs of lithium toxicity include neurologic disorders such as seizures and coma. Cardiac toxicities for lithium include ventricular arrhythmias and severe bradycardia. Common reactions seen with lithium include tremor, polyuria, diarrhea, vomiting, muscle weakness, blurred vision, and fatigue. Lithium is cleared through the kidney and dosage adjustment must be made in the setting of impaired renal function to prevent elevated levels and their associated adverse effects.

A 40-year-old man is brought to the emergency department by his friends. Apparently he has ingested some unknown medication in a suicide attempt. The patient is disoriented to time. His temperature is 39.3º C (103º F), blood pressure 120/85 mm Hg, pulse 100/min and irregular, and respirations 22/min. The skin is flushed and dry. Dilated pupils and muscle twitching are also noted. An electrocardiogram reveals that the QRS complexes are prolonged. Hepatic transaminases are normal, and blood gas analysis shows a normal pH. These findings are most likely caused by intoxication with which of the following substances? A. Acetaminophen B. Alcohol C. Benzodiazepines D. Clonidine (Catapres) E. Monoamine oxidase (MAO) inhibitors F. Tricyclic antidepressants

The correct answer is F. This patient's clinical picture is consistent with tricyclic antidepressant intoxication with amitriptyline (Elavil) or imipramine (Tofranil). Toxic effects are mediated by peripheral anticholinergic activity and "quinidine-like" action. The anticholinergic effects include mydriasis, tachycardia, impaired sweating with flushed skin, dry mouth, constipation, and muscle twitching. Quinidine-like effects (caused by block of sodium channels in the heart) result in cardiac arrhythmias, especially ventricular tachyarrhythmias. In this setting, prolongation of the QRS complex is particularly important in the diagnosis. QRS width is, in fact, an even more faithful parameter of drug toxicity than serum drug levels. In severe intoxication, patients will develop seizures, severe hypotension, and coma. Tricyclic antidepressant overdose can lead to lethal complications. The high lethality is caused by the blockage of the sodium channels, which leads to prolongation of the QT interval with subsequent torsades followed by pleomorphic ventricular tachycardia. Treatment of overdose is sodium bicarbonate, which is used to help alkalinize the urine and enhance elimination of this medication. Phenytoin (Dilantin) is used to treat cardiac arrhythmias associated with overdoses of this medication class.

A 32-year-old woman with a history of recurrent depression, borderline personality disorder, and alcohol abuse becomes pregnant. She is admitted to the crisis stabilization unit during an episode of depression with suicidal ideation after a fight with her current boyfriend and an alcohol binge. On morning rounds, the patient appears irritable. She denies being suicidal but admits to being in a bad mood and worrying about her baby. The health care provider addresses the issue of alcohol abuse and warns her about the problems related to the chronic use of alcohol during pregnancy. She gets upset about being considered an alcoholic but is willing to listen to a substance abuse counselor, who among other things describes the features of fetal alcohol syndrome. Which of the following features is associated with fetal alcohol syndrome? A. Intrauterine growth acceleration B. Large hands and feet C. Macrocephaly D. Microcephaly E. Neurofibromas

The correct answer is D. The most common clinical features of fetal alcohol syndrome (FAS) include microcephaly, short palpebral fissures, midfacial hypoplasia, and cognitive defects, which can include difficulty with attention, verbal learning, and executive function. The most common clinical features of fetal alcohol syndrome (FAS) include microcephaly, short palpebral fissures, thin upper lip, midfacial hypoplasia with flat midface, large ears, and widely spaced eyes (hypertelorism), micrognathia (undersized jaw), and small eyes. There may be long-standing cognitive defects.

A 24-year-old man presents to the emergency department with hypertension, tachycardia, elevated body temperature, diaphoresis, mydriasis, and severe agitation. His mother reports that he uses illicit drugs, although she is not sure which kind. Which of the following agents is the most appropriate therapy? A. Atropine B. Flumazenil (Romazicon) C. Fluoxetine (Prozac) D. Diazepam (Valium) E. Naloxone (Narcan)

The correct answer is D. The patient described above is probably under the influence of a CNS stimulant, such as methamphetamine or cocaine. Diazepam or another benzodiazepine are used to treat hypertension, agitation, tachycardia, and seizures. Supportive care should be given as needed to control the hyperthermia and to maintain breathing. Methamphetamine or cocaine abuse will cause symptoms consistent with sympathetic overdrive, including hyperthermia, tachycardia, mydriasis, and hypertension. Treatment of acute intoxication state would include a combined alpha- and beta-blocker to counteract the sympathetic stimulation that occurs. It would be dangerous to give a beta-blocker alone as treatment for this condition, because unopposed alpha activity would worsen the alpha-adrenergic side effects.

A 29-year-old woman is climbing the stairs in her new home when the house collapses. She is taken to the hospital with a fractured left femur. A month later the psychiatry team is consulted because the patient complains of nightmares and flashbacks and is afraid to go to sleep. During the interview she is tearful and afraid that her fear of falling is preventing her from fully participating in her rehabilitation, for which she fears the team will discharge her from the hospital. Which of the following is the most appropriate treatment? A. Insight-oriented psychotherapy B. No therapy because the patient needs to take responsibility for her treatment C. A sitter who will stay in the patient's room 24 hours a day to calm her anxiety D. Antidepressant E. Benzodiazepine

The correct answer is D. The patient is having symptoms of post-traumatic stress disorder (PTSD), as well as depressive symptoms. Sertraline (Zoloft) and paroxetine (Paxil) are SSRI antidepressants FDA-approved for treatment of this disorder. Other antidepressants and anticonvulsants have also been shown to be effective in the treatment of PTSD. The diagnosis of PTSD is made when a patient re-experiences the trauma in the form of flashbacks, intrusive memories, exaggerated emotional and physical reactions to triggers that remind the person of that trauma, and avoidance of activities, people, or places that remind the person of the trauma. Other symptoms include increased arousal in the form of difficulty sleeping and concentrating, emotional numbing, irritability, and exaggerated startle response. First-line treatment is SSRIs with the most supportive evidence for paroxetine (Paxil).

A 6-year-old boy is brought to the clinic because of poor school performance. The parents stress that the child is very hard-working and add that the child may actually work "too hard." He will only turn in work to the teacher if the margins of his writing are perfectly aligned. Additionally, the patient makes it a point to wash his hands after every paragraph that he writes. Standardized testing shows above-average intelligence and no specific weakness in an academic discipline. He has many friends and is always well-behaved and friendly in all social and academic settings. On mental status examination, he is a friendly child, appearing his stated age. He appears happy and denies any suicidal or homicidal intent, ideation, or plan. When asked why he must perform his schoolwork in this manner, the patient replies, "I don't know why. I know the other kids don't, but I just have to." There is no evidence of auditory hallucinations or disturbances in his thought process. Which of the following is the most appropriate next step in management? A. Place him in a special education setting B. Prescribe a trial of a low dose of haloperidol (Haldol) C. Prescribe a trial of methylphenidate (Ritalin) D. Recommend cognitive-behavioral therapy E. Recommend increased strictness in the patient's household

The correct answer is D. The patient is manifesting symptoms consistent with obsessive-compulsive disorder. Cognitive-behavioral therapy, including exposure and response prevention, would be of likely benefit to this patient, especially with respect to the compulsions pertaining to his schoolwork. First-line therapy for obsessive-compulsive disorder (OCD) is cognitive therapy which involves exposure and response prevention. Patients are given a hierarchy of symptom triggers and each patient is then encouraged to expose him to the trigger. The patient is then encouraged to refrain from engaging in compulsive rituals. As the patient improves, more intense triggers are targeted until the patient is able to face these triggers without having the compulsive behavior employed.

A 68-year-old woman with hypertension comes to the health care provider because of decreased appetite, fatigue, insomnia, inability to concentrate, and feelings of worthlessness over the past 2 months. She states that her grandchildren are coming to stay with her for 3 weeks and she "wants to get to the bottom of this" before they arrive. Her antihypertensive medication was changed 3 months ago. Which of the following antihypertensive medications is the most likely cause of the patient's new symptoms? A. Captopril (Catapres) B. Furosemide (Lasix) C. Minoxidil (Minoxidil) D. Propranolol (Inderal) E. Verapamil (Calan, Isoptin)

The correct answer is D. The symptoms of change in appetite, fatigue, insomnia, lack of concentration, and worthlessness for greater than 2 weeks are consistent with depression. Propranolol is one of the most common pharmacologic agents to cause depression. Other side effects of propranolol include dizziness, bronchospasm, nausea, vomiting, diarrhea, and constipation. It may also precipitate asthma, congestive heart failure, and hypoglycemia in susceptible patients. Beta-blockers that cross the blood-brain barrier have a known side effect of causing somnolence and depression-like symptoms. Bronchospasm is another side effect that occurs when patients who have reactive airways disease or asthma are placed on these agents.

The mother of a 35-year-old man calls her health care provider for advice on a drug treatment program for her son. She states that he has lived with her since the loss of his job and apartment 4 months ago. She reports that he goes out on weekends and uses drugs heavily. When he returns home, usually on Monday mornings, he sleeps for the entire day and seems very depressed. He is also very irritable and anxious over the course of the next day. She knows of no other withdrawal symptoms. She believes that he uses the drug throughout the week, but with more intense binges on the weekends. On several occasions he has called her from the local psychiatric emergency center after admitting himself for suicidal ideation after use of the drug. Which of the following drugs is most likely responsible for this man's withdrawal state? A. Alcohol B. Benzodiazepines C. Cannabis D. Cocaine E. Heroin

The correct answer is D. The use of cocaine, especially crack cocaine (an extremely potent, freebase form that is sold in small, ready-to-smoke amounts) is associated with the withdrawal symptoms described in this case. After cessation of the use of cocaine, or after acute intoxication, a post-intoxication depression (crash) often occurs. This depression is associated with dysphoria, anhedonia, anxiety, irritability, fatigue, hypersomnolence, intense craving, and suicidal ideation. Know the withdrawal symptoms for drugs that are often abused. Cocaine and crack cocaine withdrawal is characterized by dysphoria, anhedonia, anxiety, irritability, fatigue, hypersomnolence, intense craving, and suicidal ideation. Benzodiazepine withdrawal can range from tremulousness, diaphoresis, hyperreflexia, elevated blood pressure, pulse, and respiratory rates, and anxiety, to altered mental status, hallucinations, autonomic instability, seizures, and possibly death. Heroin withdrawal includes myalgias, nausea, vomiting, diarrhea, lacrimation, rhinorrhea, piloerection, anxiety, insomnia, irritability, and intense craving.

A man brings his 45-year-old wife to the emergency department. He states she has been ill for 3 days and has had a temperature 37.6-38.05° C (99.8-100.5° F). Today she is having difficulty staying awake and talking to persons who are not there. At times, she appears to be frightened of something. She is restless and somewhat combative when restrained. What is the most likely diagnosis? A. Acute stress disorder B. Bipolar I disorder, manic type C. Brief psychotic disorder D. Delirium E. Dementia

The correct answer is D. This is a psychotic level disorder (the patient is hallucinating): she has a fluctuating level of consciousness and is disoriented. Also, there is a clear history of a febrile condition that developed rather rapidly, all of which suggest delirium. Acute delirium is an acute, fluctuating change in mental status with inattention, disorganized thinking, and altered levels of consciousness. It is a potentially life-threatening medical emergency. Correction of the underlying cause is mandatory to successfully treat this condition.

A 36-year-old woman with a long history of alcoholism presents to the emergency department intoxicated. On physical examination she is disoriented and confused, and has dysarthria and oculomotor paralysis. She is unable to coordinate muscle activity during voluntary movement. Which of the following intravenous substances should most likely be administered first? A. Glucose B. Haloperidol (Haldol) C. Lorazepam (Ativan) D. Thiamine E. Valproic acid (Depakene)

The correct answer is D. This patient appears to have Wernicke encephalopathy, which is caused by a deficiency of thiamine that is quite common in chronic alcoholic patients. Symptoms of Wernicke encephalopathy include oculomotor disturbances, cerebellar ataxia, and mental confusion. Treatment consists of giving at least 3 days of thiamine, 100 mg IV or IM three times daily, along with magnesium sulfate given before loading with glucose. Patients in this situation should also be given niacin and pyridoxine (vitamin B6) since most of these patients are also deficient in these water-soluble vitamins. An alcoholic patient tends to obtain the majority of his or her caloric intake from the alcohol intake and not from nutritious intake of calories. Patients may develop Wernicke encephalopathy, which will need thiamine to prevent Korsakoff psychosis. If the patient is given a sugar load with glucose prior to giving thiamine, exacerbations and Korsakoff psychosis can occur.

A 30-year-old man attends a daily psychiatric partial-hospitalization program. At age 18, he began to hear the voices of a man and a woman telling him that he was "no good." Despite medication treatment, the patient still hears these voices 3-4 times a week. He is "used to the voices" and they do not particularly distress him any longer. He was hospitalized 2 years ago because of what he describes as a 10-day period of being "out of control." He says that at the time he had "too much energy, was spending lots of money, and talking really fast. He also started reading the Bible, which is something I never do." Which of the following is the most likely diagnosis? A. Bipolar disorder, type I B. Brief psychotic disorder C. Major depressive disorder D. Schizoaffective disorder, bipolar type E. Schizophrenia, paranoid type

The correct answer is D. This patient has a combination of a chronic psychotic illness (persistent auditory hallucinations) in addition to a clear history of a manic episode. A confusing distinction is differentiating schizoaffective disorder diagnostically from a bipolar disorder or major depression with psychotic features. This distinction is made by a required element of the diagnosis of schizoaffective disorder having psychotic symptoms in the absence of mood symptoms for at least 2 weeks. Schizoaffective disorder occurs in a patient who has longstanding psychotic features with a separate corresponding mood disorder. When a patient has a clear, new onset of a manic episode, schizoaffective disorder with bipolar type is diagnosed.

A 35-year-old woman comes to the health care provider for a periodic health maintenance examination. She has no physical complaints but she is concerned that she hardly ever feels happy. She says that she has basically been "down" for at least 3 years. She rarely goes out with friends and keeps to herself at work. Her work performance has been stable but she usually feels tired and "blah." Which of the following is the most likely diagnosis? A. Antisocial personality disorder B. Cyclothymic disorder C. Depression D. Dysthymic disorder E. Schizoaffective disorder

The correct answer is D. This patient has dysthymia, a mild, chronic form of depression. In adults, the diagnosis requires 2 years of a depressed mood, whereas in teens and adolescents it requires 1 year of symptoms to meet this criteria. Teens may manifest with irritability rather than depression. The condition may be associated with changes in appetite, sleep, and concentration, and with fatigue and hopelessness. Treatment is psychotherapy and antidepressant therapy. The primary differential diagnosis between dysthymia and major depression is the duration and severity of the disease. Patients who have dysthymia have chronic depression as their baseline with symptoms lasting at least 2 years. Patients who have major depression have an abrupt mood change from their baseline lasting at least 2 weeks with the symptoms of depression being much more significant than the chronic depressive symptoms associated with dysthymia. When major depression occurs in a patient who has dysthymia, the patient is said to have "double depression."

A 37-year-old man lives alone and has no close friends. He works during the night shift at the post office and has little interaction with others. He has not engaged in sexual activity since he was age 18, but he feels little desire. He maintains a close relationship with his sister, but he seeks out no relationships with others. People have told him that he seems "detached" and that he has difficulty experiencing or expressing emotion. Which of the following is the most likely diagnosis? A. Antisocial personality disorder B. Avoidant personality disorder C. Paranoid personality disorder D. Schizoid personality disorder E. Schizotypal personality disorder F. Social phobia

The correct answer is D. This patient has schizoid personality disorder, characterized by the inability to form relationships and difficulty experiencing and expressing emotions. Affected individuals do not seek intimacy and approval from others; they prefer to be alone and may perform well in socially isolated jobs. The incidence is thought to be very high, however it is not known, because these individuals generally do not seek help. According to the DSM, personality disorders are characterized by a stable pattern of behavior that deviates from cultural expectations, is inflexible, and causes distress and social or work impairment. Personality disorders are not caused by another medical illness or substance abuse. Schizoid personality disorder patients are grouped into cluster A classification (along with schizotypal and paranoid personality disorders). These patients enjoy loner activities. More men have this type of personality disorder than women. Personality disorders have a chronic pattern of both internal perception and observable behavior. Patients who have personality disorders are most likely to come to the attention of medical providers when they are treated for another comorbid condition such as depression.

A 26-year-old medical student is arrested for threatening a neighbor with a knife. He is brought to the hospital because he tried to slit his wrists on the way to the police station. His roommate arrives and states that the patient has been "very odd" over the past 7 months. He is socially withdrawn and has been overheard saying that the FBI is giving him tips on patient care and that "people from the sky" are telling him to do "bad things." He stopped going to his Ob-Gyn rotation because he believes that the babies were "coming to earth" to give him messages. He says he does not want to be a doctor anymore. He appears apathetic and has emotional blunting. Physical examination is otherwise normal. Toxicologic screening is negative. Which of the following is the most likely diagnosis? A. Bipolar I disorder B. Brief psychotic disorder C. Delusional disorder D. Schizophrenia E. Schizophreniform disorder

The correct answer is D. This patient has schizophrenia. He hears voices, has delusional irrational thoughts, has a flattened affect and lack of motivation, and has become socially withdrawn. This has been going on for 7 months. Schizophrenia is a disease that is characterized by psychotic symptoms (positive symptoms) and flattening of affect and motivation (negative symptoms) and an impairment of social or occupational functioning for at least 6 months. The symptoms are not caused by a mood disorder, a schizoaffective disorder, or a medical, neurologic, or a substance-induced disorder. It commonly begins in the early 20s, and an abnormal premorbid level of functioning is common. Ten percent of patients will commit suicide. Treatment consists of antipsychotic agents and psychosocial treatment, including reality-based therapy, patient and family support, skill training, and assistance with daily living skills. Schizophrenia is a chronic mental illness characterized by psychosis, disorganized speech, negative symptoms (demotivation, self-neglect, reduced emotion), and positive symptoms (auditory hallucinations, thought disorder, delusions) that last at least 6 months of this functioning decline. Antipsychotic medications and psychotherapy are effective for most patients, although compliance with treatment is difficult to maintain.

A 25-year-old man comes to a mental health care provider after being referred by his primary care provider for evaluation of possible depression. During the interview, the patient states that he lives alone and works as the film projectionist at a local movie theater. He has no outside interests, but claims that he is comfortable living alone and has not had any changes in appetite or sleep. He describes how he ignores social invitations from some of his co-workers and spends most of his leisure time watching television and reading comic books. Which of the following is the most likely diagnosis? A. Antisocial personality disorder B. Borderline personality disorder C. Dependent personality disorder D. Schizoid personality disorder E. Schizophrenia

The correct answer is D. This patient has symptoms of schizoid personality disorder, characterized by restricted emotions and social detachment. Patients are distant from other individuals, and in contrast to those who have avoidant personality disorder, do not desire contact with other individuals in social situations. They usually have a very narrow range of interests. Schizoid personality disorder patients are grouped into cluster A classification (along with schizotypal and paranoid personality disorders). These patients enjoy loner activities. More men have this type of personality disorder than women. Personality disorders have a chronic pattern of both internal perception and observable behavior. Patients who have personality disorders are most likely to come to the attention of medical providers when they are treated for another comorbid condition such as depression.

An 18-year-old girl comes to the health care provider for a "checkup" before going off to college. She has no complaints. She states that she is very excited to finally be on her own. She exercises "regularly," gets good grades in school, and is sexually active. Her weight is normal for her height. Physical examination shows many dental caries, periodontal disease, pharyngeal abrasions, nail changes, and multiple, linear lacerations on her dorsal wrists in various stages of healing. Laboratory studies show hypokalemia and metabolic alkalosis. Which of the following would most likely establish a diagnosis? A. "Do you ever feel guilty about drinking alcohol?" B. "Do you typically restrict your diet to under 800 calories a day?" C. "Have you ever taken laxatives as a way to lose weight?" D. "Have you ever consumed large quantities of food and then vomited in order to prevent weight gain?" E. "Please describe your exercise routine."

The correct answer is D. This patient most likely has bulimia nervosa. It is a condition characterized by recurrent episodes of binge eating followed by a compensatory behavior to prevent weight gain (vomiting, exercise, laxative abuse). Other features include stealing (food), alcohol and drug abuse, self-mutilation, and depression. The individuals are usually at or slightly over the normal weight for their height, sexual activity is normal or increased, and they continue to menstruate. Clinical findings that are caused by recurrent vomiting include dental caries, periodontal disease, pharyngeal lacerations, and nail changes. Metabolic alkalosis and hypokalemia are also present. Complications include aspiration and rupture of the esophagus or stomach. Bulimia nervosa is an eating disorder that is characterized by preoccupation with weight and body shape. This condition includes recurrent episodes of binge eating with compensatory mechanisms such as self-induced vomiting, excessive exercise, or use of laxatives to prevent weight gain. Binges are associated with feelings of lack of control over intake of food. Women predominate with this condition and patients are typically not thin. Physical manifestations may include parotid hypertrophy and erosion of enamel of teeth. Patients may also have Russell sign, which is scarring over the dorsum of the hands from self-induced vomiting. Patients who have this condition may also have self-injurious behavior.

A 72-year-old African American woman is being readied for discharge from the hospital 2 weeks after a stroke affected her right occipital cortex. A health care provider working in psychiatry is called for consultation to evaluate the patient for depression, as she has had decreased appetite, some crying spells, and insomnia. After determining that the patient is not suffering from delirium, which of the following medications would be most appropriate to treat her depressive symptoms? A. Methylphenidate (Ritalin) B. Nortriptyline (Pamelor) C. Phenelzine (Nardil) D. Sertraline (Zoloft) E. Thioridazine (Mellaril)

The correct answer is D. With complications from stroke, an SSRI such as sertraline is the safest and most effective medication for the treatment of clinical depression. SSRIs are considered to be first-line therapy for major depressive episodes because of its proven effectiveness and low side-effect profile. Tricyclics and MAO inhibitors are associated with significant side effects, which relegates them to second-line therapy status.

A 34-year-old woman with documented bipolar disorder comes to the hospital because of feelings of guilt and worthlessness. She has amassed thousands of dollars of credit card debt in the past 2 months. Her parents are going to pay it off for her, but she believes that this will bankrupt them. On further questioning she reveals that she bought a gun earlier in the day because it would be easier for everyone if she "wasn't here anymore." She plans to go to the roof of her building with her new purchase "and do what needs to be done." Which of the following is the most appropriate next step in management? A. Send home with lithium B. Send home with valproic acid (Depakene) C. Send home with carbamazepine (Tegretol) D. Observe in the emergency department E. Admit to the hospital

The correct answer is E. Anyone who has serious suicidal thoughts, suicidal intent, and a plan, must be hospitalized, against her will if necessary. The patient expressed a desire to die and she bought a weapon and developed a believable plan. She needs to be hospitalized for her own safety. Patients with bipolar have a lifetime suicide rate of 10-15%. Patients who have mood disorders or other psychiatric disease may go on to attempt or commit suicide. Health care providers need to be vigilant on assessing patients for suicide thoughts or actual plans and will need to admit these patients (voluntarily or involuntarily) to the hospital to observe them and to provide treatment to lessen their attempt at suicide.

A 17-year-old girl is found unconscious in a bathroom after ingesting a large amount of unknown sleeping pills. On arriving at the emergency department, she is somnolent; her temperature is 37.2º C (98.9º F), blood pressure is 120/74 mm Hg, pulse is 68/min, and respirations are 12/min. She responds to painful stimuli and to hearing her name loudly by opening her eyes briefly. She returns to the somnolent state immediately afterward, however. Which of the following is the most appropriate initial intervention? A. IV administration of caffeine B. IV administration of droperidol (Inapsine) C. IV administration of 500 mL of normal saline D. Ipecac-induced emesis E. Gastric lavage

The correct answer is E. Gastric lavage should be used in this somnolent patient. It is best done within the first 60 minutes of ingestion of the unknown substance. It is performed by inserting a large-caliber orogastric tube and then flushing the tube with a large amount of normal saline. The saline is then aspirated from the stomach to a container in an attempt to evacuate any ingested substance that persists in the stomach cavity. Identification of the ingested substance is also very important. With certain ingested substances, gastric lavage is contraindicated. For example, the procedure should not be used if hydrocarbon ingestion is suspected because aspiration is much more likely to happen during a lavage or an induced emesis. Furthermore, ingestion of alkaline substances causes esophageal necrosis. In this setting, gastric lavage may cause esophageal perforation. Acute drug overdose needs to be managed emergently. Gastric lavage should be attempted if the drug overdose is of recent onset and the patient is symptomatic as a result of this drug ingestion. Airway management should be a priority during this procedure and the patient should be closely observed to prevent any aspiration of gastric contents when this procedure is used.

A 42-year-old HIV-positive man is being treated for his first episode of Pneumocystis jirovecii pneumonia on an inpatient medical ward when he begins to become increasingly disoriented, combative, and agitated. Pulse oximetry shows that he is oxygenating at 93% on room air. He is currently receiving nortriptyline (Pamelor) for depression and gabapentin (Neurontin) for pain caused by peripheral neuropathy. He has a history of alcohol abuse for the past several years before hospitalization and a remote history of opioid abuse. Which of the following is the most likely diagnosis? A. Alcohol withdrawal B. HIV dementia C. Hypoxemic encephalopathy D. Major depressive disorder E. Multifactorial delirium

The correct answer is E. In a patient such as this who has a medical illness, a history of the acute onset of agitation, disorientation, and confusion is consistent with and most suggestive of the diagnosis of delirium. Delirium is a medical emergency. It is characterized by an acute fluctuating change in mental status with inattention, disorganized thinking, and altered level of consciousness. There is a high morbidity and mortality associated with this disorder. Risk factors for delirium include new onset neurologic illness, myocardial infarction, severe systemic infection (UTI and pneumonia in older people), respiratory disorders (hypoxia and pulmonary embolism), alcohol abuse, hip fracture, metabolic abnormalities (sodium, potassium, and calcium), glucose abnormalities (hypoglycemia and hyperglycemia), drug toxicity (anticholinergics, tricyclics, stimulants), drug withdrawal states, acute psychosis, malignant disease, urinary outlet obstruction and endocrine abnormalities. Treatment is correction of the underlying cause which led to the episode of delirium.

A health care provider is performing a mental status examination on a 78-year-old man who was brought to the clinic because of recent cognitive decline. The health care provider believes that the patient's presentation is consistent with Alzheimer's disease. The examination includes a question which asks the patient what he would do if he found someone else's social security check mixed in with his own mail at home. Which of the following mental status testing is being assessed? A. Abstract thinking B. Cognition C. Insight D. Intelligence E. Judgment

The correct answer is E. Judgment is the ability of a patient to evaluate a particular course of action to determine if it is the most appropriate one within the patient's particular value system. If the patient were to say he would put the letter back in the mail or give it back to the postman, this would indicate appropriate judgment. Different aspects of a patient's mental status are ascertained during formal mental status testing. Judgment assesses whether a patient can logically determine the most appropriate action during a proposed situation. Abstract thinking assesses the patient's ability to interpret proverbs. Cognition is tested by assessing a patient's intellectual knowledge. Insight is assessed by asking patients to recognize the reality of a situation. Calculation and concentration is assessed by serial 7 subtraction from 100 or asking the patient to spell the word "world" backward.

A patient is brought to a mental health professional after being persuaded by her family to get help. She describes that she thought she was depressed. She feels empty and guilty for not having any feelings for her children. She notices that mornings are the worst. Later in the day, she is able to do some things. In the mornings, however, she stays in bed late and, even though she has never been lazy, she is unable to do anything. She falls asleep quickly at night, but wakes up at 4 AM and is unable to get back to sleep. She is sure there isn't anything that could cheer her up, even for a short time. Which of the following is the most likely diagnosis? A. Catatonia B. Cotard syndrome C. Dysthymia D. Major depression with atypical features E. Major depression with melancholic features

The correct answer is E. Major depression with melancholic features is characterized by a depressed mood most of the time and a lack of reactivity to pleasurable stimuli during episodes. In addition, three or more of the following criteria must be met: distinct quality of depressed mood, worse depression in the morning, early morning awakening, psychomotor retardation or agitation, weight loss, and inappropriate guilt. Major depressive disorder is a period of at least 2 weeks during which time the patient has persistent low mood, anhedonia, and reduced energy. Neurovegetative disturbance is commonly encountered with the patient having early morning awakening, difficulty with sleep initiation, poor concentration, and poor appetite.

A 25-year-old moderately obese woman with a history of bipolar disorder diagnosed at age 19 complains to her health care provider that her recent struggles with her weight and eating have caused her to feel depressed. She states that she has recently had increased difficulty sleeping, has felt excessively anxious and agitated, and has had increasing thoughts of suicide. She often finds herself fidgety and unable to sit still for extended periods of time. Her family tells her that she is increasingly irritable. She has never attempted suicide. Her current medications include lithium carbonate (Lithobid) and zolpidem (Ambien) as needed for sleep. Which of the following is more consistent with a diagnosis of major depressive disorder than bipolar depression in this patient during this episode? A. Early onset of bipolar disorder B. Increased appetite C. Insomnia D. Increased thoughts of suicide E. Psychomotor agitation

The correct answer is E. Major depressive disorder is a syndrome that requires a depressed mood or loss of interest or pleasure and causes clinically significant distress or impairment in occupational, social, or other important areas of functioning. The diagnosis of bipolar depression requires that the patient have also had a manic episode of bipolar disorder. Bipolar depression is associated with less psychomotor agitation compared with unipolar depression (major depressive disorder), and thus the patient's agitation is more consistent with a major depressive disorder diagnosis than with the depression of bipolar disorder. It is challenging to differentiate between major depressive episode and the depression that can occur as part of bipolar disease. Patients who have pure major depression typically will have more psychomotor agitation. Typical symptoms of major depression include difficulty concentrating and sleeping, weight change, and appetite change. Psychomotor agitation occurs with anxiety and restlessness.

A 32-year-old man with a history of chronic paranoid schizophrenia is brought to his health care provider after failing courses of treatment with both haloperidol (Haldol) and olanzapine (Zyprexa). His health care provider would like to start him on clozapine (Clozaril) but she is concerned about the patient's ability to get back to the clinic on a regular basis for monitoring. Which of the following parameters should the psychiatrist monitor carefully in this patient? A. Blood pressure B. Platelet count C. Pulse D. Respiratory rate E. White blood cell count

The correct answer is E. Of all the indicators listed, white blood cell count should concern the health care provider most, as there is a documented risk for agranulocytosis associated with the use of the antipsychotic clozapine. Most patients on clozapine require regular monitoring of the CBC to prevent the risk for severe infection associated with agranulocytosis. There is a black box warning with clozapine for agranulocytosis such that this medication has restricted use due to this life-threatening complication. Monitoring parameters include initial CBC at baseline, during therapy, and up until 4 weeks after discontinuation of this medication.

A grandmother brings her 13-year-old grandson, who is in her custody, to a health care provider who works in psychiatry. He has been talking back to his teachers and not complying with school rules. When asked to stop, he gets angry and blames others, usually his classmates. In addition, his grades are deteriorating. The grandmother has been struggling with him in the past year and thinks he needs "structure and control." The boy says she doesn't let him do anything and he resents having to live with her. Psychologic testing shows no indication of a learning disorder. Which of the following is the most likely diagnosis? A. Adjustment disorder with disturbance of conduct B. Attention deficit/hyperactivity disorder C. Conduct disorder D. Cornelia de Lange syndrome E. Oppositional defiant disorder

The correct answer is E. Oppositional defiant disorder belongs to the group of disruptive behaviors characterized by a pattern of negativistic hostile behavior lasting at least 6 months and including 4+ of the following: arguing with others, blaming others for one's own mistakes, being angry and resentful, being vindictive, refusing to comply with adults' requests, and being easily annoyed by others. The disorder is not caused by a mood or psychotic disorder. It causes significant impairment in functioning. Patients who have oppositional defiant disorder will have persistent patterns of tantrums, arguing, and disruptive behavior toward authority figures. Children who have this disorder become overtly uncooperative and hostile in a way that is much more dramatic and serious than others in the matching age cohort. Affected patients have overt defiance and refusal to comply with adult requests. Some children who have this disorder may go on to develop conduct disorder.

A 50-year-old female politician has an "extreme, paralyzing fear" before walking to a podium to give a speech. She is so anxious about these situations that it is beginning to impair her professional activities. She is terrified of stuttering and "making a fool" of herself in front of unfamiliar people. Which of the following is the most likely diagnosis? A. Agoraphobia B. Generalized anxiety disorder (GAD) C. Panic disorder D. Posttraumatic stress disorder (PTSD) E. Social phobia

The correct answer is E. This patient has a social phobia (performance anxiety). Social phobia is a fear of social or performance situations in which the individual may be exposed to unfamiliar individuals or may be evaluated by others. It is a common disorder that affects men and women. It impairs everyday activities and relationships. Blushing, shaking, hypertension, and tachycardia occur when the individual is exposed to the feared situation. Beta-blockers are effective in preventing the symptoms. Social phobia or social anxiety disorder is a condition in which patients have fears over being embarrassed and negatively evaluated by others. Symptoms of distress or impairment have to be present at least 6 months to meet the criteria. Treatment involves medication, psychotherapy, or a combination of the two.

A 15-year-old boy is referred to a mental health professional by his school counselor and is brought in by his mother for increasing hostility and argumentativeness with his teachers and toward his parents over the last school year. He often loses his temper and often blames his classmates when his misbehavior causes him to receive punishment in class. He had done well previously with his schoolwork. He has no history of theft, destruction of property, or physical violence. He hangs around with other boys his age that have similar behavior. Which of the following is the most likely diagnosis? A. Attention deficit/hyperactivity disorder (ADHD) B. Conduct disorder C. Dysthymic disorder D. Mental retardation E. Oppositional defiant disorder

The correct answer is E. Oppositional defiant disorder is a pattern of negativistic, hostile, or defiant behavior lasting at least 6 months, characterized by losing one's temper, arguing with adults, and the deliberate annoyance of people or blaming of others for mistakes or misbehavior. It occurs in the absence of conduct disorder (choice B), which is characterized by the breaking of age-appropriate social norms, with associated destruction of property, physical violence, and theft. Oppositional defiant disorder is characterized by children and teenagers displaying openly hostile, disobedient, and defiant behavior. Conduct disorder is characterized by behavior that involves destruction of property, physical violence, or theft. Societal norms of behavior are broken with conduct disorder, whereas oppositional defiant disorder is more about attitude than behavior.

A 50-year-old woman with a past history of recurrent major depressive disorder is currently on a maintenance dose of an antidepressant. She has been well for the past 2 years. She is complaining to her mental health provider about her decreased ability to reach orgasm. Which of the following medications has most likely caused her anorgasmia? A. Amitriptyline (Elavil) B. Bupropion (Buspar) C. Mirtazapine (Remeron) D. Nefazodone (Serzone) E. Paroxetine (Paxil)

The correct answer is E. Paroxetine, along with other selective serotonin reuptake inhibitors, can cause decreased libido and difficulties reaching orgasm. In studies, the incidence of these side effects was 2-9% in patients on paroxetine compared with those on placebo. Major depression is typically treated with SSRIs as first-line therapy. Although these agents have a good safety profile (especially when compared to tricyclic antidepressants), they do have some side effects: suicidality, serotonin syndrome, hyponatremia, priapism, first trimester teratogenicity, and sexual side effects such as decreased libido, anorgasmia, and ejaculatory dysfunction.

A 48-year-old woman with a history of depression has been maintained on phenelzine (Nardil) for several years. She comes to the health care provider complaining about the worsening of her depression and insists on switching to a newer medication such as an SSRI. She is also "sick of sticking to such a rigid diet." Which of the following statements is correct concerning switching her from phenelzine (Nardil) to an SSRI A. Abrupt switching may cause hypotension B. A 4-week washout after discontinuing phenelzine (Nardil) is needed before starting an SSRI C. It is safe to switch right away D. Stopping phenelzine (Nardil) causes discontinuation syndrome E. A 10-day washout period after discontinuing phenelzine (Nardil) is needed before an SSRI can be started

The correct answer is E. Serotonin syndrome can develop with concomitant use of MAO inhibitors and serotonergic agents, or if there has been too short a period of washout in crossing over from one to another agent. At least 10 days after MAO inhibitor discontinuation and 5 weeks after discontinuation of fluoxetine (Prozac) should pass to avoid this possible complication. Clinical presentation includes tremor, hypertension, confusion, incoordination, fever, myoclonus, diarrhea, restlessness, and diaphoresis. Serotonin syndrome can result if withdrawal from MAO inhibitors prior to starting an SSRI is not performed correctly. Both MAO inhibitors and SSRIs are associated with serotonin syndrome, so there needs to be a washout from one medication to another to lessen this serotonin toxicity. Serotonin syndrome presentation typically consists of neuromuscular excitation, autonomic effects, and altered mental status. Patients may have hyperreflexia, clonus, tremor, diaphoresis, headaches, or sweating. There may also be tachycardia, anxiety, and agitation with a fever. Diagnosis is clinical since no lab tests can make this diagnosis. The condition can be avoided by carefully withdrawing one medication prior to starting another one. Treatment is cessation of the medication, IV fluids to treat hypotension, and chlorpromazine (Thorazine) IV as an antagonist to serotonin.

A 32-year-old man with no past history of mental health treatment has "occupational problems." He feels that he has not advanced professionally in his job as a financial analyst; however, he is able to perform above expectations in the work that he performs privately. He is "paralyzed by fear" when he gives presentations at work. He is also unable to concentrate and develops palpitations around strangers at parties for work and social occasions. Which of the following is the most likely diagnosis? A. Generalized anxiety disorder B. Major depressive disorder C. Obsessive-compulsive disorder D. Post-traumatic stress disorder E. Social phobia

The correct answer is E. Social phobia is an anxiety disorder characterized by a patient fearing situations in which he or she is the object of attention. Many patients who have social phobia are, like this patient, unable to advance professionally because of professional demands including public interaction. Social phobia is characterized by an excessive fear of social and performance situations in which the patient is afraid of being embarrassed or negatively evaluated by others. SSRIs are first-line medical therapy, along with cognitive behavioral therapy. Relapse for this type of phobia is common.

A 36-year-old married woman who works in a nursing home presents to the office and complains of chest pain and pressure. She has had similar episodes in the past, and evaluations have always been normal. At the time of the initial workup, she told the health care provider that she has been a "worrier" since childhood and is still quite anxious, and the health care provider told her that she has a "nervous heart." Despite these normal evaluations in the past, she still believes she has serious heart disease. Since puberty, she suffered from dysmenorrhea, nausea, vomiting, and fatigue, and she "finally" had a hysterectomy with bilateral salpingo-oophorectomy 5 years ago because of severe menorrhagia, which was "the last straw." She suffers from an "irritable colon" and a "weak stomach." Last year she had an episode of dizziness and unstable gait that lasted for "some time," but a workup done at the time was negative. She suffers from migraine headaches and the standard migraine medications do not help. She "does not even bother" going to a regular health care provider for her back pain, but she sees a chiropractor on a regular basis, which brings some relief. She has been hospitalized 7 times in the past 5 years for many of the abovementioned reasons. Which of the following is the most likely diagnosis? A. Conversion disorder B. Generalized anxiety disorder C. Masked depression D. Pain disorder E. Somatization disorder

The correct answer is E. Somatization disorder belongs to the category of somatoform disorder. It is characterized by the following characteristics: multiple medical complaints resulting in significant diagnostic testing and medical interventions with symptoms causing impaired social and occupational functioning. These symptoms cannot be explained by medical findings. Symptoms include pain in at least 4 sites: 1 pseudoneurologic symptom, 1 sexual dysfunction symptom, and at least 2 or more GI symptoms. Somatization disorder is a non-intentionally produced condition that is commonly seen in primary care practice. Multiple body systems are involved. Symptoms cannot be explained from a physiologic standpoint and multiple tests reveal no organic cause. Despite the negative workup of symptoms, patients are not reassured. Risk factors for this condition include female sex and a history of sexual abuse.

A mental health care provider on a consult team is asked to see a 32-year-old woman for depression. She was admitted to the hospital for a workup of nausea, vomiting, and abdominal pain. The medical team has so far found no reason for these symptoms. In talking to the patient, the mental health care provider learns that she has had many medical problems and, over the past 10 years, has been in the hospital more than 10 times. She reports having had excessive menstrual bleeding and a miscarriage, as well as difficulty becoming aroused during sexual intercourse. She cannot tolerate high-fiber foods and has heartburn and bloating. Two years ago, she had a lump in her throat and had difficulty swallowing, but nothing was found during that admission. In the past couple of years, she has had frequent migraine headaches and blurred vision and has seen several neurologists. She also hurt her back and now has chronic back pain. Which of the following is the most likely diagnosis? A. Conversion disorder B. Generalized anxiety disorder C. Masked depression D. Pain disorder E. Somatization disorder

The correct answer is E. Somatization disorder is characterized by multiple medical complaints resulting in significant diagnostic testing and medical interventions and causing impaired social and occupational functioning. The symptoms, which cannot be explained by medical findings, include pain in at least 4 sites, one pseudoneurologic symptom, one sexual dysfunction symptom, and at least two or more GI symptoms. Somatization disorder is a non-intentionally produced condition that is commonly seen in primary care practice. These symptoms cannot be explained from a physiologic standpoint and multiple tests reveal no organic cause or explanation for these symptoms. Despite these symptoms not having a readily explainable cause, patients are not reassured of their benign nature regardless of the negative workup of these symptoms. If the patient is reassured that this is a benign constellation of symptoms, the symptoms typically will abate with time. Risk factors for this condition include female sex and having a history of sexual abuse. Multiple body systems are involved with somatization disorder.

A 28-year-old anesthesiology resident has been having a hard time in his demanding residency because of the back pain he experiences after long hours of duty. He was involved in a motor vehicle accident 4 years ago and suffered a serious back injury. One day his wife, who is also a physician, comes home early to find him in bed, wrapped in a blanket complaining of fever, chills, and runny nose. He complains of body aches and soon afterward develops diarrhea and cramps. The wife confronts him about possible drug use and he admits that he is "finally trying to quit for good." Based on these symptoms, he is most likely experiencing withdrawal from which of the following substances? A. Alprazolam (Xanax) B. Amphetamine C. Cocaine D. Nicotine E. Opioid

The correct answer is E. Symptoms of opioid withdrawal follow cessation or reduction of prolonged and heavy opioid use and include at least 3 of the following criteria: dysphoric mood, nausea or vomiting, muscle aches, yawning, diarrhea, lacrimation or rhinorrhea, piloerection, pupillary dilatation, sweating, fever, and insomnia. The symptoms cause significant distress in social or occupational functioning and are not caused by a general medical condition. Opioid withdrawal occurs if the dose of opioids is abruptly decreased or discontinued. This withdrawal state can occur within a few hours after the last opioid is taken. Symptoms of withdrawal include agitation, restlessness, and muscle and bone pain. Insomnia, vomiting and diarrhea, chills with goose bumps, and restless legs can also occur with opiate withdrawal. Detoxification of opiate addiction is performed with buprenorphine (Subutex, Buprenex), which binds to opioid receptors, and nalaxone (Narcan).

A 55-year-old woman was diagnosed with schizophrenic disorder, paranoid type at age 23. She has been on neuroleptic medications for the majority of time since diagnosis. Which of the following conditions would be most likely to appear at this time as a consequence of her treatment? A. Akathisia B. Anticholinergic symptoms C. Dystonia of the laryngeal muscles D. Neuroleptic malignant syndrome E. Tardive dyskinesia

The correct answer is E. Tardive dyskinesia is a result of long-term treatment with neuroleptic medications and is seen more frequently in older women. The condition presents with abnormal choreoathetoid movements, often involving the face and mouth in adults and the limbs in children. Once tardive dyskinesia develops, there is no known treatment to mitigate the symptoms. In order to prevent this long-term sequela of tardive dyskinesia, patients may be placed on drug holidays or are placed on the smallest effective dose in order to control symptoms. Long-term use of antipsychotics can lead to several medication-related side effects. Tardive dyskinesia is more likely to occur with the use of the typical antipsychotics, which are known to cause more extrapyramidal side effects. Tardive dyskinesia consists of choreoform movements. Once this side effect occurs, it is irreversible and tends to be slowly progressive. The mouth and tongue are most commonly affected by these movements.

A 31-year-old woman is brought to a local police precinct by a pedestrian because she appeared confused. She does not know how she arrived in this town. She has had other discrete episodes of being in cities, not knowing how she arrived. Additionally, she has a history of finding clothes in her apartment that she does not remember buying and hotel receipts in her pockets with another woman's signature.Which of the following factors is most likely present in this patient's history? A. Cardiac disease B. Food allergies C. Lead exposure D. Neuroleptic malignant syndrome E. Physical or sexual abuse

The correct answer is E. The patient most likely has dissociative identity disorder. There is a high correlation between physical and sexual abuse and dissociative disorders. For patients who have dissociative identity disorder, the fragmenting of a distinct and integrated personality into multiple personalities may have the adaptive purpose of protecting the more vulnerable and frightened facets of the core personality after abusive trauma. Dissociative disorder involves a period of detachment from self or surrounding experience as being "unreal" or "outside" of the person or self. This condition is associated with periods of time that the person cannot recall. Fugue states with dissociative disorder involve a person who leaves a geographic area and assumes another personality that is not consciously produced or occurs without knowledge of various personalities occurring within the person. Dissociative identity disorder is usually precipitated by a stressful situation and it is theorized that the alter identity is formed to escape from the stressful situation. Repression and isolation are other coping mechanisms to deal with this stress. Sexual abuse can be a precipitating event associated with dissociative disorder.

A 30-year-old woman has had a history since adolescence of multiple vague physical complaints, including headaches, nausea, bloating, abdominal pain, dysmenorrhea, fatigue, fainting, and dysuria. She is now being seen in a health care provider's office, where she describes her current abdominal symptoms as "the worst imaginable." On physical examination no abdominal stiffness or masses are noted. Tenderness on palpation of the abdomen is elicited, but the site varies and is not reproducible, even a minute or so later. On talking with the patient, she does not seem concerned about any specific life events. Which of the following is the most likely diagnosis? A. Conversion disorder B. Hypochondriasis C. Malingering D. Munchausen syndrome E. Somatization disorder

The correct answer is E. This patient has somatization disorder. The physical complaints are usually vague and not adequately explained by a physical disorder. Formal criteria for the diagnosis have been defined and include onset before age 30, symptoms involving at least 4 different body parts, two or more GI symptoms, at least one reproductive or sexual symptom, and at least one neurologic symptom other than pain. Patients may have coexistent personality disorders, notably histrionic, borderline, and antisocial. Treatment is difficult, and the condition tends to wax and wane throughout life. Suicide is a definite risk with overtly depressed patients who have longstanding disease. Somatoform disorders include both conversion disorders and somatization. With somatoform disorders, patients experience distressing physical symptoms that are not fully explained by other medical or psychiatric disorders. These symptoms occur from psychologic stress that is unconsciously expressed in a physical way. Risk factors include being female, having been abused or at least having a lot of childhood difficulties, and difficulty expressing emotions. These symptoms can lead to significant functional impairment and distress. Diagnosis is made after evaluating the patient for an explainable physical cause. Treatment is primarily done with cognitive behavioral therapy.

A 16-year-old boy is brought to the clinic by his father who says that he has been increasingly aggressive and has been stumbling and tripping around the house for several weeks. He is especially worried about his uncharacteristic violent behavior. The patient's temperature is 38º C (100.4º F), blood pressure 140/90 mm Hg, pulse 90/ min, and respirations 22/min. He has slightly dilated pupils and nystagmus. He begins to have convulsions on the table, making the remainder of the examination impossible. Which of the following is the most likely cause of these findings? A. Cocaine withdrawal B. Heroin C. Marijuana D. Morphine E. Phencyclidine (PCP)

The correct answer is E. This patient is displaying the signs and symptoms of PCP intoxication. PCP causes aggression, distortion of body image, disorganized thoughts, ataxia, nystagmus, mid-dilated pupils, myoclonus, fever, hypersalivation, and hyperacusis. It can lead to seizures, coma, and death. It can also lead to an acute psychosis with a high risk for violent behavior and suicide. Initial therapy for an agitated or psychotic state is diazepam (Valium), whereas persistent psychotic states may need antipsychotics such as haloperidol (Haldol). PCP intoxication can result in hostile and aggressive behavior. Patients under acute intoxication may have high blood pressure along with dilated and nonreactive pupils.

A medical consult is requested for a 32-year-old patient in a closed psychiatric unit. The patient suffers from paranoid schizophrenia. Several days after admission into the unit the patient developed polyuria, vomiting, stupor, diarrhea, and restlessness. She currently takes risperidone (Risperdal) 10 mg at bedtime, but no other medications. She has no other medical conditions. Which of the following is the most likely diagnosis? A. Acute dystonic reaction B. Anticholinergic crisis C. Serotonin syndrome D. Tardive dyskinesia E. Water intoxication

The correct answer is E. This patient is showing the symptoms of psychogenic polydipsia, which is the excessive intake of water as a result of a psychiatric disorder. The symptoms of excessive water intake include polyuria, vomiting, diarrhea, difficulty concentrating, confusion, and lethargy. As the patient is on a closed psychiatric unit, the chances of reactions to excessive medications are rare, given the careful monitoring of medication intake on most psychiatric units. Surreptitious water consumption would be easy to overlook in a patient who has no previous history. Risperidone use can lead to abdominal pain, nausea, and vomiting. Psychogenic polydipsia is the excessive intake of water in a patient who has a psychiatric disorder. Patients with this condition will develop hyponatremia as a result of this inappropriately increased water intake. Surreptitious water consumption may be easy to overlook in a patient presenting with polyuria, confusion, and vomiting.

A 37-year-old man with chronic paranoid schizophrenia is being interviewed by a 4th-year medical student. In the middle of describing how he feels that other people are out to kill him, the patient suddenly stops talking and begins staring into space. Approximately 20 seconds later, he starts talking about how his mother was not a very good parent and was excessive in her discipline during his childhood. Which of the following terms best describes this behavior? A. Echolalia B. Echopraxia C. Mutism D. Satyriasis E. Thought blocking

The correct answer is E. Thought blocking is the sudden repression of anxiety-provoking thoughts in mid-sentence. It is a common finding in many individuals who have schizophrenia. Often when the patient begins speaking again, it is concerning something unrelated to the topic before the pause in conversation and thought. Thought blocking is a term used to describe a process by which a patient starts a conversation, stops, and then restarts the conversation about a totally unrelated topic. It is a feature that can occur in schizophrenia and some other psychotic disorders. Patients may also have verbigeration, in which they have a repetition of words in the absence of stimuli, and perserveration, which is a repetition of the same response to different stimuli.

A 20-year-old man is brought to the clinic by his parents, who are concerned that he is an alcoholic. They want him to check himself in for treatment. In a discussion about his drinking habits, the young man brags that he can now drink more before he gets drunk. Which of the following terms best characterizes this phenomenon? A. Blackout B. Confabulation C. Dereism D. Resistance E. Tolerance

The correct answer is E. Tolerance describes the phenomenon of a drinker needing greater amounts of alcohol to get the same effect. It develops over time and is an indication of dependence. Certain medications (such as narcotics) or alcohol have a phenomenon known as tolerance associated with their use. Tolerance is the reduced effect of alcohol that occurs when a body is used to having alcohol. Chronic tolerance is caused by induction of alcohol-metabolizing enzymes. Experience with alcohol can result in extremely high blood alcohol levels without the person showing signs of intoxication.


Set pelajaran terkait

Productions 2 Exam- Queuing Theory, Operations Management: Ch 7, Chapter 10, QUEUING THEORY, Operations Management, Operations Management: Ch 10

View Set

Bio Ch 15- Mechanisms of evolution

View Set

Chapter 2 Financial statements, taxes, cash flows

View Set

Organizational Behavior Day 1 & Day 2

View Set